NEET 2015 Previous Year Paper

NEET 2015

Section

Questions

Marks

Physics

45 Questions (1 – 45)

180

Chemistry

45 Questions (46 – 90)

180

Biology

90 Questions (91 – 180)

360

Q. 1 If force (F ), velocity (V) and time (T) are chosen as fundamental quantities, the dimensional formula of surface tension will be

A. [E V^(−2) T^(−1)]

B. [E V^(−1) T^(−2)]

C. [E V^(−2) T^(−2)]

D. [E^(-2) V^(−1) T^(−3)]

 

Q. 2 A Ship A is moving westwards with a speed of 10 km h⁻¹ and a ship B 100 km south of A is moving northwards with a speed of 10 km h⁻¹. The time after which the distance between them becomes shortest is

A. 0 h

B. 5 h

C. 5√2 h

D. 10√2 h

 

Q. 3 A particle of unit mass undergoes one-dimensional motion such that its velocity varies according to υ(x)=βx-2n where β and n are constants and x is the position of the particle. The acceleration of the particle as a function of x is given by

A. -2 n β^2 x^(-2n-1)

B. -2 n β^2 x^(-4n-1)

C. -2 n β^2 x^(-2n+1)

D. -2 n β^2 x^(-4n+1)

 

Q. 4 Three blocks A, B and C, of masses 4 kg, 2 kg and 1 kg, respectively, are in contact on a frictionless surface, as shown. If a force of 14 N is applied on the 4 kg block, then the contact force between A and B is

A. 2 N

B. 6 N

C. 8 N

D. 18 N

 

Q. 5 A block A of mass m1 rests on a horizontal table. A light string connected to it passes over a frictionless pulley at the edge of table and from its other end another block B of mass m2 is suspended. The coefficient of kinetic friction between the block and the table is μk. When the block is sliding on the table, the tension in the string is

A. (m2 +μk m1 )g / (m1 + m2)

B. (m2 -μk m1 )g / (m1 + m2)

C. (m2 . m2(μk + 1))g / (m1 + m2)

D. (m2 . m2(1 – μk))g / (m1 + m2)

 

Q. 6 Two similar springs P and Q have spring constants KP and KQ. They are stretched, first by the same amount (case a), then y the same force (case b). The work done by the springs WP and WQ are related as in case (a) and case (b), respectively:

A. WP = WQ; WP > WQ

B. WP = WQ; WP = WQ

C. WP > WQ; WQ > WP

D. WP < WQ; WQ < WP

 

Q. 7 A block of mass 10 kg, moving in the x-direction with a constant speed of 10 ms⁻¹ subjected to a retarding force F = 0.1x J/m during its travel from x = 20 m to 30 m. final KE will be

A. 475 J

B. 450 J

C. 275 J

D. 250 J

 

Q. 8 A particle of mass m is driven by a machine that delivers a constant power k watts. If the particle starts from rest the force on the particle at time t is

A. (√mk / 2) t⁻¹/²

B. (√mk ) t⁻¹/²

C. (√2mk ) t⁻¹/²

D. 1 / (√mk ) t⁻¹/²

 

Q. 9 Two particles of masses m1, m2 move with initial velocities u1 and u2. On collision, one of the particles get excited to higher level, after absorbing energyε . If final velocities of particles be v1 and v2 then we must have

A. m1^(2) u1 +m2^(2) u2 -ε = m1^(2) v1 +m2^(2) v2

B. 1/2 (m1^(2) u1 +m2^(2) u2 ) = 1/2 (m1^(2) v1 +m2^(2) v2) – ε

C. 1/2 (m1 u1^(2) +m2 u2^(2) ) – ε = 1/2 (m1^ v1^(2) +m2 v2^(2))

D. 1/2 (m1^(2) u1 +m2^(2) u2 ) – ε = 1/2 (m1^(2) v1 +m2^(2) v2)

 

Q. 10 The rod of weight W is supported by two parallel knife edges A and B and is in equilibrium in a horizontal position. The knives are at a distance d from each other. The centre of mass of the rod is at distance x from A. The normal reaction on A is

A. Wx / d

B. Wd / x

C. W(d – x) / x

D. W(d – x) / d

 

Q. 11 A mass m moves in a circle on a smooth horizontal plane with velocity v0 at a radius R0. The mass is attached to a string which passes through a smooth hole in the plane as shown. The tension in the string is increased gradually and finally m moves in a circle of radius R0. The final value of the kinetic energy is

A. m v^2 R0

B. 1/4 m v^2 R0

C. 2 m v^2 R0

D. 1/2 m v^2 R0

 

Q. 12 Three identical spherical shells, each of mass m and radius r placed as shown in figure. Consider an axis XX’ which is touching to two shells and passing through diameter of third shell. Moment of inertia of the system consisting of these three spherical shells about XX’ axis is

A. 11/5 m r^2

B. 3 m r^2

C. 16/5 m r^2

D. 4 m r^2

 

Q. 13 Kepler’s third law states that square of period of revolution (T) of a planet around the Sun is proportional to third power of average distance r between Sun and planet i.e. T^2 = K r^3 here K is constant. If the masses of Sun and planet are M and m, respectively, then as per Newton’s law of gravitation, force of attraction between them is F = GMm/ r^2, here G is the gravitational constant. The relation between G and K is described as

A. GK = 4 π^2

B. GMK = 4 π^2

C. K = G

D. K = 1/G

 

Q. 14 Two spherical bodies of mass M and 5 M and radii R and 2 R are released in free space with initial separation between their centres equal to 12 R. If they attract each other due to gravitational force only, then the distance covered by the smaller body before collision is

A. 2.5 R

B. 4.5 R

C. 7.5 R

D. 1.5 R

 

Q. 15 On observing light from three different starts P, Q and R, it was found that intensity of violet colour is maximum in the spectrum of P, the intensity of green colour is maximum in the spectrum of R and the intensity of red colour is maximum in the spectrum of Q. If TP, TQ and TR are the respective absolute temperatures of P, Q and R, then it can be concluded from the above observation that

A. TP > TQ > TR

B. TP > TR > TQ

C. TP < TR < TQ

D. TP < TQ < TR

 

Q. 16 16. The approximate depth of an ocean is 2700 m. the compressibility of water is 45.4 x 10^(−11) Pa^(−1) and density of water is 10^3 kg/m^3. What fractional compression of water will be obtained at the bottom of the ocean?

A. 0.8 x 10^(-2)

B. 1.0 x 10^(-2)

C. 1.2 x 10^(-2)

D. 1.4 x 10^(-2)

 

Q. 17 The two ends of a metal rod maintained at temperatures 100 ⁰C and 110 ⁰C. The rate of heat flow in the rod is found to be 4.0 J/s. If the ends are maintained at temperatures 200 ⁰C and 210 ⁰C, the rate of heat flow will be

A. 44.0 J/s

B. 16.8 J/s

C. 8.0 J/s

D. 4.0 J/s

 

Q. 18 A wind with speed 40 m/s blows parallel to the roof of a house. The area of the roof is 250 m2. Assuming that the pressure inside the house is atmospheric pressure, the force exerted by the wind on the roof and the direction of the force will be (P air = 1.2 kg/m3)

A. 4.8 x 10^5 N, downwards

B. 4.8 x 10^5 N, upwards

C. 2.4 x 10^5 N, upwards

D. 2.4 x 10^5 N, downwards

 

Q. 19 Figure below shows two paths that may be taken by a gas to go from a state A to a state C. In process AB, 400 J of heat is added to the system and in process BC, 100 J of heat is added to the system. The heat absorbed by the system in the process AC will be

A. 380 J

B. 500 J

C. 460 J

D. 300 J

 

Q. 20 A Carnot engine having an efficiency of η=1/10 as heat engine is used as a refrigerator. If the work done on the system is 10 J, the amount of energy absorbed from the reservoir at lower temperature is

A. 100 J

B. 99 J

C. 90 J

D. 1 J

 

Q. 21 One mole of an ideal diatomic gas undergoes a transition from A to B along a path AB as shown in the figure,The change in internal energy of the gas during the transition is

A. 20 kJ

B. – 20 kJ

C. 20 J

D. -12 kJ

 

Q. 22 The ratio of the specific heats Cp / Cy =γ in terms of degrees of freedom (n) is given by

A. 1 = 1/3

B. 1 + n/3

C. 1 + 2/n

D. 1+ n/2

 

Q. 23 When two displacements represented by y1 = a sin (ωt) and y2 = b cos (ωt) are

superimposed the motion is

A. Not a simple harmonic

B. Simple harmonic with amplitude a/b

C. Simple harmonic with amplitude √( a^2 + b^2 )

D. Simple harmonic with amplitude (a + b)/2

 

Q. 24 A particle is executing SHM along a straight line. Its velocities at distances x1 and x2 from the mean position are V1 and V2, respectively. Its time period is

A. 2π √( (x1^2 + x2^2)/(v1^2 + v2^2) )

B. 2π √( (x2^2 – x1^2)/(v1^2 – v2^2) )

C. 2π √( (v1^2 + v2^2)/(x1^2 + x2^2) )

D. 2π √( (v1^2 – v2^2)/(x1^2 – x2^2) )

 

Q. 25 The fundamental frequency of a closed organ pipe of length 20 cm is equal to the second overtone of an organ pipe open at both the ends. The length of organ pipe open at both ends is

A. 80 cm

B. 100 cm

C. 120 cm

D. 140 cm

 

Q. 26 A parallel plate air capacitor of capacitance C is connected to a cell of emf V and then disconnected from it. A dielectric slab of dielectric constant K which can just fill the air gap of the capacitor is now inserted in it. Which of the following is incorrect?

A. The potential difference between the plates decrease K times

B. The energy stored in the capacitor decreases K times

C. The change in energy stored is 1/2 ( C V^2)( 1/K – 1)

D. The charge on the capacitor is not conserved

 

Q. 27 The electric field in a certain region is acting radially outward and is given by E = Ar. A charge contained in a sphere of radius ‘a’ centred at the origin of the field, will be given by

A. 4π ε0 Aa^2

B. ε0 Aa^2

C. 4π ε0 Aa^3

D. ε0 Aa^3

 

Q. 28 A potentiometer wire has length 4 m and resistance 8 W. The resistance that must be connected in series with the wire and an accumulator of emf 2 V, so as to get a potential gradient 1 mV per cm on the wire is

A. 32 Ω

B. 40 Ω

C. 44 Ω

D. 48 Ω

 

Q. 29 A, B and C are voltmeters of resistance R, 1.5R and 3R, respectively, as shown in the figure. When some potential difference is applied between X and Y, the voltmeter readings are VA, VB and VC, respectively, then

A. VA = VB = VC

B. VA ≠ VB = VC

C. VA = VB ≠ VC

D. VA ≠ VB ≠ VC

 

Q. 30 Across a metallic conductor of non-uniform cross section a constant potential difference is applied. The quantity which remains constant along the conductor is

A. current density

B. current

C. drift velocity

D. electric field

 

Q. 31 A wire carrying current I has the shape as shown in adjoining figure. Linear parts of the wire are very long and parallel to X-axis, while semicircular portion of radius R is lying in Y–Z plane. Magnetic field at point O is

A. B = (μ0 I / 4πR) ( πi + 2k )

B. B = – (μ0 I / 4πR) ( πi – 2k )

C. B = -(μ0 I / 4πR) ( πi + 2k )

D. B = (μ0 I / 4πR) ( πi – 2k )

 

Q. 32 An electron moving in a circular orbit of radius r makes n rotations per second. The magnetic field produced at the centre has magnitude:

A. μ0 n e / 2πr

B. zero

C. μ0 n 2 e

D. μ0 r n e / 2r

 

Q. 33 A conducting square frame of side ‘a’ and a long straight wire carrying current I are located in the same plane as shown in the figure. The frame moves to the right with a constant velocity ‘V’. The emf induced in the frame will be proportional to

A. 1 / x^2

B. 1 / (2x – a)^2

C. 1 / (2x + a)^2

D. 1 / (2x + a)(2x – a)

 

Q. 34 A resistance ‘R” draws power ‘P’ when connected to an AC source. if an inductance is now placed in series with the resistance, such that the impedance of the circuit becomes ‘Z’, the power drawn will be

A. P ( R/Z )^2

B. P√(R/Z)

C. P ( R/Z )

D. P

 

Q. 35 A resistance ‘R’ draws power ‘P’ when connected to an AC source. If an inductance is now placed in series with the resistance, such that the impedance of the circuit becomes ‘Z’, the power drawn will be

A. P ( R/Z)^2

B. P √(R/Z)

C. P ( R/Z)

D. E / C^2

 

Q. 36 Two identical thin plano–convex glass lenses (refractive index 1.5) each having radius of curvature of 20 cm are placed with their convex surfaces in contact at the centre. The intervening space is filled with oil of refractive index 1.7. The focal length of the combination is

A. -20 cm

B. -25 cm

C. -50 cm

D. 50 cm

 

Q. 37 For a parallel beam of monochromatic light of wavelength ‘λ’, diffraction is produced by a single slit whose width ‘a’ is of the order of the wavelength of the light. If ‘D’ is the distance of the screen from the slit, the width of the central maxima will be

A. 2Dλ/a

B. Dλ/a

C. Da/λ

D. 2Da/λ

 

Q. 38 In a double slit experiment, the two slits are 1 mm apart and the screen is placed 1 m away. A monochromatic light of wavelength 500 nm is used. What will be the width of each slit for obtaining ten maxima of double slit within the central maxima of single slit pattern?

A. 0.2 mm

B. 0.1 mm

C. 0.5 mm

D. 0.02 mm

 

Q. 39 The refracting angle of a prism is A, and refractive index of the material of the prism is cot (A/2). The angle of minimum deviation is

A. 180° – 3A

B. 180° – 2A

C. 90° – A

D. 180°+ 2A

 

Q. 40 A certain metallic surface is illuminated with monochromatic light of wavelength λ. The stopping potential for photoelectric current for this light is 3V0. If the same surface is illuminated with light of wavelength 2λ, the stopping potential is V0. The threshold wavelength for this surface for photoelectric effect is

A. 6λ

B. 4λ

C. 4^λ

D. 6^λ

 

Q. 41 Which of the following figures represent the variation of particle momentum and the associated de-Broglie wavelength?

A. (1)

B. (2)

C. (3)

D. (4)

 

Q. 42 Consider 3rd orbit of He+ (Helium), using non-relativistic approach, the speed of electron in this orbit will be [given K = 9 x 10^9 constant, Z = 2 and h(Planck’s constant) = 6.6 x 10^−34 Js]

A. 2.92 x 10^6 m/s

B. 1.46 x 10^6 m/s

C. 0.73 x 10^6 m/s

D. 3.0 x 10^6 m/s

 

Q. 43 If radius of the 27/13 Al nucleus is taken to be R(Al) then the radius of 125/53 Te nucleus is nearly:

A. (53/13)^(1/3) R

B. 5/3 R

C. 3/5 R

D. 13/53 R

 

Q. 44 If in a p–n junction, a square input signal of 10 V is applied, as shown

A. (1)

B. (2)

C. (3)

D. (4)

 

Q. 45 Which logic gate is represented by the following combination of logic gates?

A. OR

B. NAND

C. AND

D. NOR

 

Q. 46 Which of the following species contains equal number of σ and π -bonds?

A. HCO₃⁻

B. XeO₄

C. CN₂

D. CH₂ CN₂

 

Q. 47 The species Ar, K+ and Ca2+ contain the same number of electrons. In which order do their radii increase?

A. Ar < K+ < Ca2+

B. Ca2+ < Ar <K+

C. Ca2+ < K+ < Ar

D. K+ < Ar < Ca2+

 

Q. 48 The function of ‘’Sodium pump’’ is a biological process operating in each and every cell of all animals. Which of the following biologically important ions is also a constituent of this pump?

A. Ca2+

B. Mg2+

C. K+

D. Fe2+

 

Q. 49 ‘’Metals are usually not found as nitrates in their ores’’. Out of the following two (a and b) reasons which is/are true for the above observation?(a)Metal nitrates are highly unstable (b) Metal nitrates are highly soluble in water

A. a and b are true

B. a and b are false

C. a is false but b is true

D. a is true but b is false

 

Q. 50 Solubility of the alkaline earth metal sulphates in water decreases in the sequence :

A. Mg > Ca > Sr > Ba

B. Ca > Sr > Ba > Mg

C. Sr > Ca > Mg > Ba

D. Ba > Mg > Sr > Ca

 

Q. 51 Because of lanthanoid contraction, which of the following pairs of elements have nearly same atomic radii? (Numbers in the parenthesis are atomic numbers).

A. Ti (22) and Zr (40)

B. Zr (40) and Nb (41)

C. Zr (40) and Hf (72)

D. Zr (40) and Ta (73)

 

Q. 52 Which of the following processes does not involve oxidation of iron?

A. Rusting of iron sheets

B. Decolourisation of blue CuSO₄ solution by iron

C. Formation of FeCO₅ from Fe

D. Liberation of H² from steam by iron at high temperature

 

Q. 53 Which of the following pairs of ions are isoelectronic and isostructural?

A. CO₃²⁻ , SO₃²⁻

B. ClO₃²⁻ , CO₃²⁻

C. SO₃²⁻ , NO₃⁻

D. ClO₃⁻ , SO₃²⁻

 

Q. 54 Which of the following options represents the correct bond order?

A. O₂⁻ >O₂ >O₂⁺

B. O₂⁻ <O₂ <O₂⁺

C. O₂⁻ >O₂ <O₂⁺

D. O₂- <O₂ >O₂⁺

 

Q. 55 Nitrogen dioxide and sulphur dioxide have some properties in common. Which property is shown by one of these compounds, but not by the other?

A. forms ‘acid – rain’

B. is a reducing agent

C. is soluble in water

D. is used as a food-preservative

 

Q. 56 Maximum bond angle at nitrogen is present in which of the following

A. NO₂

B. NO₂⁻

C. NO₂⁺

D. NO₃⁻

 

Q. 57 Magnetic moment 2.84 B.M. is given by: (At. nos. Ni = 28, Ti = 22, Cr = 24, Co = 27)

A. Ni₂⁺

B. Ti₃⁺

C. Cr₂⁺

D. Co₂⁺

 

Q. 58 Cobalt (III) Chloride forms several octahedral complexes with ammonia. Which of the following will not give test for chloride ions with silver nitrate at 25°C?

A. CoCl₃ . 3NH₃

B. CoCl₃ . 4NH₃

C. CoCl₃ . 5NH₃

D. CoCl₃ .6NH₃

 

Q. 59 Which of these statements about [Co(CN)₆]³⁻ is true?

A. [[Co(CN)₆]³⁻ has no unpaired electrons and will be in a low-spin configuration

B. [Co(CN)₆]³⁻ has four unpaired electrons and will be in low-spin configuration

C. [Co(CN)₆]³⁻ has four unpaired electrons and will be in a high-spin configuration

D. [Co(CN)₆]³⁻ has no unpaired electrons and will e in a high-spin configuration

 

Q. 60 The activation energy of a reaction can be determined from the slope of which of the following graphs?

A. In K vs. T

B. InK/T vs.T

C. InKvs. 1/T

D. T/InK vs. 1/T

 

Q. 61 Which one is not equal to zero for an ideal solution?

A. ΔHmix

B. ΔSmix

C. ΔVmix

D. ΔP=Pobserved -PRaoult

 

Q. 62 A mixture of gases contains H2 and O2 gases in the ratio of 1:4 (w/w). What is the molar ratio of the two gases in the mixture?

A. 1:4

B. 4:1

C. 16:1

D. 2:1

 

Q. 63 A given metal crystallizes out with a cubic structure having edge length of 361 pm. If there are four metal atoms in one unit cell, what is the radius of one atom?

A. 40 pm

B. 127 pm

C. 80 pm

D. 108 pm

 

Q. 64 When initial concentration of a reactant is doubled in a reaction, its half – life period is not affected. The order of the reaction is :

A. Zero

B. First

C. Second

D. More than zero but less than first

 

Q. 65 If the value of an equilibrium constant for a particular reaction is 1.6 × 1012, then at equilibrium the system will contain

A. All reactants

B. Mostly reactants

C. Mostly products

D. Similar amounts of reactants and products

 

Q. 66 A device that converts energy of combustion of fuels like hydrogen and methane, directly into electrical energy is known as :

A. Fuel cell

B. Electrolytic cell

C. Dynamo

D. Ni–Cd cell

 

Q. 67 The boiling point of 0.2 mol kg^−1 solution of X in water is greater than equimolal solution of Y in water. Which one of the following statements is true in this case?

A. X is undergoing dissociation in water

B. Molecular mass of X is greater than the molecular mass of Y

C. Molecular mass of X is less than the molecular mass of Y

D. Y is undergoing dissociation in water, while X undergoes no change

 

Q. 68 Which one of the following electrolytes has the same value of van’t Hoff’s factor (i) as that of Al₂(SO₄)₃ (if all are 100% ionized)?

A. K₂SO₄

B. K₃[Fe(CN)₆]

C. Al(NO₃)₃

D. K₄[Fe(CN)₆]

 

Q. 69 The number of d-electrons in Fe2+ (Z = 26) is not equal to the number of electrons in which one of the following?

A. s-electrons in Mg (Z = 12)

B. p-electrons in Cl (Z = 17)

C. d-electrons in Fe (Z = 26)

D. p-electrons in Ne (Z = 10)

 

Q. 70 The correct bond order in the following species is:

A. O²⁺₂ < O₂⁺ < O₂⁻

B. O²⁺₂ <O₂⁻ < O₂⁺

C. O₂⁺ < O₂⁻ < O²⁺₂

D. O₂⁻ < O₂⁺ < O²⁺₂

 

Q. 71 The angular momentum of electron in ‘d’ orbital is equal to:

A. √ 6 h

B. √2 h

C. 2 √3 h

D. 0 h

 

Q. 72 The Ksp of Ag₂CrO₄, AgCl, AgBr and AgI are, respectively, 1.1 × 10⁻¹², 1.8 × 10⁻¹⁰, 5.0 × 10⁻¹³, 8.3 × 10⁻¹⁷. Which one of the following salts will precipitate last if AgNO₃ solution is added to the solution containing equal moles of NaCl, NaBr, NaI and Na₂CrO₄

A. AgI

B. AgCl

C. AgBr

D. AgCrO₄

 

Q. 73 Which property of colloidal solution is independent of charge on the colloidal particles?

A. Coagulation

B. Electrophoresis

C. Electro-osmosis

D. Tyndall effect

 

Q. 74 Which of the following statements is correct for a reversible process in a state of equilibrium?

A. ΔG=-2.30RTlogK

B. ΔG=2.30RTlogK

C. ΔGo =-2.30RTlogK

D. ΔGo =2.30RTlogK

 

Q. 75 Bithional is generally added to the soaps as an additive to function as a/an:

A. Softener

B. Dryer

C. Buffering agent

D. Antiseptic

 

Q. 76 The electrolytic reduction of nitrobenzene in strongly acidic medium produces:

A. P-Aminophenol

B. Azoxybenzene

C. Azobenzene

D. Aniline

 

Q. 77 In Duma’s method for estimation of nitrogen 0.25 g of an organic compound gave 40 mL if nitrogen collected at 300 K temperature and 725 mm pressure. If the aqueous tension at 300 K is 25 mm, the percentage of nitrogen in the compound is:

A. 17.36

B. 18.20

C. 16.76

D. 15.76

 

Q. 78 In which of the following compounds, the C–Cl bond ionization shall give most stable carbonium ion?

A. (1)

B. (2)

C. (3)

D. (4)

 

Q. 79 The given reaction is

A. Williamson Synthesis

B. Williamson continuous etherification process

C. Etard reaction

D. Gatterman–Koch reaction

 

Q. 80 The reaction of C6H5CH = CHCH3 with HBr produces

A. (1)

B. (2)

C. (3)

D. (4)

 

Q. 81 A single compound of the given structure is obtainable from ozonolysis of which of the following cyclic compounds

A. (1)

B. (2)

C. (3)

D. (4)

 

Q. 82 Treatment of cyclopentanone the following species?

A. Cyclopentanonyl anion

B. Cyclopentadienyl cation

C. Cyclopentadienyl radical

D. Cyclopentanonyl biradical

 

Q. 83 Consider the following compounds. The hyperconjugation occurs in?

A. I only

B. II only

C. III only

D. I and III

 

Q. 84 Which of the following is the most correct electron displacement of a nucleophilic reaction to take place

A. (1)

B. (2)

C. (3)

D. (4)

 

Q. 85 The enolic form of ethyl acetoacetate as below has:

A. 18 sigma bonds and 2 pi-bonds

B. 16 sigma bonds and 1 pi-bonds

C. 9 sigma bonds and 2 pi-bonds

D. 9 sigma bonds and 1 pi-bonds

 

Q. 86 Given, Which of the given compounds can exhibit tautomerism?

A. I and II

B. I and III

C. II and III

D. I, II and III

 

Q. 87 The enthalpy of hydrogenation of these compounds will be in the order as:

A. I > II > III

B. III > II > I

C. II > III > I

D. II > I > III

 

Q. 88 Biodegradable polymer which can be produced from glycine and aminocaproic acid is

A. Nylon 2–nylon 6

B. PHBV

C. Buna-N

D. Nylon 6, 6

 

Q. 89 The total number of π- bond electrons in the following structure is:

A. 4

B. 8

C. 12

D. 16

 

Q. 90 An organic compound ‘X’ having molecular formula C5H10O yields phenyl hydrazone and gives negative response to the iodoform test and Tollens’ test. It produces n-pentane on reduction. ‘X’ could be:

A. pentanal

B. 2-pentanone

C. 3-pentanone

D. n-amyl alcohol

 

Q. 91 Which one of the following matches is correct?

(1)

Phytophthora 

Aseptate mycelium

Basidiomycetes 

(2)

Alternaria 

Sexual reproduction absent

Deuteromycetes 

(3)

Mucor 

Reproduction by conjugation

Ascomycetes 

(4)

Agaricus 

Parasitic fungus 

Basidiomycetes 

A. (1)

B. (2)

C. (3)

D. (4)

 

Q. 92 Read the following five statements (A to E) and select the option with all correct statements: (A) Mosses and Lichens are the first organisms to colonise a bare rock (B) Selaginella is a homosporous pteridophyte. (C) Coralloid roots in Cycas have VAM (D) Main plant body in bryophytes is gametophytic, whereas in pteridophytes is sporophytic (E) In gymnosperms, male and female gametophytes are present within sporangia located on sporophyte.

A. (A),(C) AND (D)

B. (B),(C) and (D)

C. (A),(D) and (E)

D. (B),(C) and (E)

 

Q. 93 In which of the following gametophyte is not independent free living

A. Funaria

B. Marchantia

C. Pteris

D. Pinus

 

Q. 94 Which one of the following statements is wrong?

A. Algin and carrageenan are products of algae

B. Agar–agar is obtained from Gelidium and Gracilaria

C. Chlorella and Spirulina are used as space food

D. Mannitol is stored food in Rhodophyceae

 

Q. 95 The guts of cow and buffalo possess

A. Fucus spp.

B. Chlorella spp.

C. Methanogens

D. Cyanobacteria

 

Q. 96 Male gametes are flagellated in

A. Polysiphonia

B. anabaena

C. Ectocarpus

D. Spirogyra

 

Q. 97 Vascular bundles in monocotyledons are considered closed because

A. A bundle sheath surrounds each bundle

B. Cambium is absent

C. There are no vessels with perforations

D. Xylem is surrounded all around by phloem

 

Q. 98 The given figure is the floral image of

 

A. Allium

B. Sesbania

C. Petunia

D. Brassica

 

Q. 99 A major characteristic of the monocot root is the presence of

A. Open vascular bundles

B. Scattered vascular bundles

C. Vasculature without cambium

D. Cambium sandwiched between phloem and xylem long the radius

 

Q. 100 Keel is the characteristic feature of flower of

A. Tulip

B. Indigofera

C. Aloe

D. Tomato

 

Q. 101 Perigynous flowers are found in

A. Guava

B. Cucumber

C. China rose

D. Rose

 

Q. 102 Leaves become modified into spines in

A. Opuntia

B. Pea

C. Onion

D. Silk Cotton

 

Q. 103 The structures that are formed by stacking of organised flattened membranous sacs in the chloroplasts are

A. Cristae

B. Grana

C. Stroma lamellae

D. Stroma

 

Q. 104 The chromosomes in which centromere is situated close to one end are

A. Metacentric

B. Acrocentric

C. Telocentric

D. Sub-metacentric

 

Q. 105 Select the correct matching in the following pairs

A. Smooth R – Oxidation of phospholipids

B. Smooth ER – Synthesis of lipids

C. Rough ER – Synthesis of glycogen

D. Rough ER – Oxidation of fatty acids

 

Q. 106 True nucleus is absent in

A. Anabaena

B. Mucor

C. Vaucheria

D. Volvox

 

Q. 107 Which one of the following is not an inclusion body found in prokaryotes?

A. Phosphate granule

B. Cyanophycean granule

C. Glycogen granule

D. Polysome

 

Q. 108 Transpiration and root pressure cause water to rise in plants by

A. Pulling it upward

B. Pulling and pushing it, respectively

C. Pushing it upward

D. Pushing and pulling it, respectively

 

Q. 109 Minerals known to be required in large amounts for plant growth include

A. Phosphorus, potassium, sulphur, calcium

B. Calcium, magnesium, manganese, copper

C. Potassium, phosphorus, selenium, boron

D. Magnesium, sulphur, iron, zinc

 

Q. 110 What causes a green plant exposed to the light on only one side to bend toward the source of light as it grows?

A. Green plants need light to perform photosynthesis

B. Green plants seek because they are phototropic

C. Light stimulates plant cells on the lighted side to grow faster

D. Auxin accumulates on the shaded side, stimulating greater cell elongation there.

 

Q. 111 In a ring girdled plant

A. The shoot dies first

B. The root dies first

C. The shoot and root die together

D. Neither root nor shoot will die

 

Q. 112 Typical growth curve in plants is

A. Sigmoid

B. Linear

C. Stair–steps shaped

D. Parabolic

 

Q. 113 Which one gives the most valid and recent explanation for stomatal movements?

A. Transpiration

B. Potassium influx and efflux

C. Starch hydrolysis

D. Guard cell photosynthesis

 

Q. 114 The hilum is a scar on the

A. Seed, where funicle was attached

B. Fruit, where it was attached to pedicel

C. Fruit, where style was present

D. Seed, where micropyle was present

 

Q. 115 Which one of the following may require pollinators, but is genetically similar to autogamy?

A. Geitonogamy

B. Xenogamy

C. Apogamy

D. Cleistogamy

 

Q. 116 Which one of the following statement is not true?

A. Pollen grains are rich in nutrients, and they used in the form of tablets and syrups

B. Pollen grains of some plants cause severe allergies and bronchial afflictions in some people

C. The flowers pollinated by flies and bats secrete four odour to attract them

D. Honey is made by bees by digesting pollen collected from flowers

 

Q. 117 Transmission tissue is characteristics feature of

A. Hollow style

B. Solid style

C. Dry stigma

D. Wet stigma

 

Q. 118 In ginger vegetative propagation occurs through

A. Rhizome

B. Offsets

C. Bulbils

D. Runners

 

Q. 119 Which of the following are the important floral rewards to the animal pollinators?

A. Colour and large size of flower

B. Nectar and pollen grains

C. Floral fragrance and calcium crystals

D. Protein pellicle and stigmatic exudates

 

Q. 120 How many pairs of contrasting characters in pea plants were studied by Mendel in his experiments?

A. Five

B. Six

C. Seven

D. Eight

 

Q. 121 Which is the most common mechanism of genetic variation in the population of a sexually reproducing organism?

A. Transduction

B. Chromosomal aberrations

C. Genetic drift

D. Recombination

 

Q. 122 A technique of micropropagation is

A. Somatic hybridisation

B. Somatic embryogenesis

C. Protoplast fusion

D. Embryo rescue

 

Q. 123 The movement of a gene from one linkage group to another is called

A. Inversion

B. Duplication

C. Translocation

D. Crossing over

 

Q. 124 Multiple alleles are present

A. On different chromosomes

B. At different loci on the same chromosome

C. At the same locus of the chromosome

D. On non-sister chromatids

 

Q. 125 Which body of the Government of India regulates GM research and safety of introducing GM organisms for public services?

A. Biosafety committee

B. Indian council of agricultural research

C. Genetic Engineering Approval committee

D. Research committee on Genetic Manipulation

 

Q. 126 In BT cotton, the BT toxin present in plant tissue as pro-toxin is converted into active toxin due to

A. Alkaline pH of the insect gut

B. Acidic pH of the insect gut

C. Action of gut microorganisms

D. Presence of conversion factors in insect gut

 

Q. 127 The crops engineered for glyphosate are resistant/tolerant to

A. Fungi

B. Bacteria

C. Insects

D. Herbicides

 

Q. 128 DNA is not present in

A. Chloroplast

B. Ribosomes

C. Nucleus

D. Mitochondria

 

Q. 129 Which of the following enhances or induces fusion of protoplasts?

A. Sodium chloride and potassium chloride

B. Polyethylene glycol and sodium nitrate

C. IAA and kinetin

D. IAA and gibberellins

 

Q. 130 The UN conference of Parties on climate change in the year 2011 was held in

A. Poland

B. South Africa

C. Peru

D. Qatar

 

Q. 131 Vertical distribution of different species occupying different levels in a biotic community is known as

A. Divergence

B. Stratification

C. Zonation

D. Pyramid

 

Q. 132 In which of the following both pairs have correct combination?

A. In situ conservation : National Park Ex situ conservation : Botanical Garden

B. In situ conservation : Cryopreservation Ex situ conservation : Wildlife sanctuary

C. In situ conservation : Seed Bank Ex situ conservation : National Park

D. In situ conservation : Tissue culture Ex situ conservation : Sacred groves

 

Q. 133 Secondary succession takes place on/in

A. Bare rock

B. Degraded forest

C. Newly created pond

D. Newly cooled lava

 

Q. 134 The mass of living material at a trophic level at a particular time is called

A. Gross primary productivity

B. Standing state

C. Net primary productivity

D. Standing crop

 

Q. 135 In an ecosystem the rate of production of organic matter during photosynthesis is termed

A. Net primary productivity

B. Gross primary productivity

C. Secondary productivity

D. Net productivity

 

Q. 136 Which of the following characteristics is mainly responsible for diversification of insects on land?

A. Segmentation

B. Bilateral symmetry

C. Exoskeleton

D. Eyes

 

Q. 137 Which of the following endoparasites of humans does show viviparity

A. Ancylostoma duodenale

B. Enterobius vermicularis

C. Trichinella spiralis

D. Ascaris lumbricoides

 

Q. 138 Which of the following represents the correct combination without any exception?

A. (1)

B. (2)

C. (3)

D. (4)

 

Q. 139 Which of the following animals is not viviparous?

A. Flying fox (bat)

B. Elephant

C. Platypus

D. Whale

 

Q. 140 Erythropoiesis starts in

A. Kidney

B. Liver

C. Spleen

D. Red bone marrow

 

Q. 141 The terga, sterna and pleura of cockroach body are joined by

A. Cementing glue

B. Muscular tissue

C. Arthrodial membrane

D. Cartilage

 

Q. 142 Nuclear envelope is a derivative of

A. Smooth endoplasmic reticulum

B. Membrane of Golgi complex

C. Microtubules

D. Rough endoplasmic reticulum

 

Q. 143 Cytochromes are found in

A. Matrix of mitochondria

B. Outer wall of mitochondria

C. Cristae of mitochondria

D. Lysosomes

 

Q. 144 Which one of the following statements is incorrect?

A. A competitive inhibitor reacts reversibly with the enzyme to form an enzyme– inhibitor complex

B. In competitive inhibition, the inhibitor molecule is not chemically changed by the enzyme

C. The competitive inhibitor does not affect the rate breakdown of the enzyme– substrate complex

D. The presence of the competitive inhibitor decreases the Km of the enzyme for the substrate

 

Q. 145 Select the correct option for (a) (b) (c) (d)

A. (ii) (i) (iii) (iv)

B. (ii) (iii) (v) (iv)

C. (i) (ii) (v) (iv)

D. (ii) (iii) (iv) (v)

 

Q. 146 A somatic cell that has just completed the S phase of its cell cycle, as compared to gamete of the same species, has

A. Twice the number of chromosomes and twice the amount of DNA

B. Same number of chromosomes but twice the amount of DNA

C. Twice the number of chromosomes and four times the amount of DNA

D. Four times the number of chromosomes and twice the amount of DNA

 

Q. 147 Which of the following statements is not correct?

A. Brunner’s glands are present in the submucosa of stomach and secrete pepsinogen

B. Goblet cells are present in the mucosa of intestine and secrete mucus

C. Oxyntic cells are present in the mucosa of stomach and secrete HCl

D. Acini are present in the pancreas and secrete carboxypeptidase

 

Q. 148 Gastric juice of infants contains

A. Maltase, pepsinogen, rennin

B. Nuclease, pepsinogen, lipase

C. Pepsinogen, lipase, rennin

D. Amylase, rennin, pepsinogen

 

Q. 149 When you hold your breath, which of the following gas changes in blood  would first lead to the urge to breathe?

A. Falling O₂ concentration

B. Rising CO₂ concentration

C. Falling CO₂ concentration

D. Rising CO₂ and falling O₂ concentration

 

Q. 150 Blood pressure in the mammalian aorta is maximum during

A. Systole of the left atrium

B. Diastole of the right ventricle

C. Systole of the left ventricle

D. Diastole of the right atrium

 

Q. 151 Which one of the following is correct?

A. Plasma = Blood – Lymphocytes

B. Serum = Blood + Fibrinogen

C. Lymph = Plasma + RBC + WBC

D. Blood = Plasma + RBC + WBC + Platelets

 

Q. 152 Removal of proximal convoluted tubule from the nephron will result in

A. More diluted urine

B. More concentrated urine

C. No change in quality and quantity of urine

D. No urine formation

 

Q. 153 Sliding filament theory can best explained as

A. When filaments slide past each other actin filaments shorten, while Myosin filament do not shorten

B. Actin and Myosin filaments shorten and slide pass each other

C. Actin and Myosin filaments shorten and slide pass each other

D. When myofilaments slide pass each other, Myosin filaments shorten, while Actin filaments do not shorte

 

Q. 154 Glenoid cavity articulates

A. Clavicle with acromion

B. Scapula with acromion

C. Clavicle with scapula

D. Humerus with scapula

 

Q. 155 Which of the following regions of the brain is incorrectly paired with its function?

A. Medulla oblongata – Homoeostatic control

B. Cerebellum – Language comprehension

C. Corpus callosum – Communication between the left and right cerebral cortices

D. Cerebrum – Calculation and contemplation

 

Q. 156 A gymnast is able to balance his body upside down even in the total darkness because of

A. Cochlea

B. Vestibular apparatus

C. Tectorial membrane

D. Organ of Corti

 

Q. 157 A chemical signal that has both endocrine and neural roles is

A. Melatonin

B. Calcitonin

C. epinephrine

D. Cortisol

 

Q. 158 Which of the following does not favour the formation of large quantities of dilute urine?

A. Alcohol

B. Caffeine

C. Renin

D. Atrial-natriuretic factor

 

Q. 159 Capacitation refers to changes in the

A. Sperm before fertilisation

B. Ovum before fertilisation

C. Ovum after fertilisation

D. Sperm after fertilisation

 

Q. 160 Which of these is not an important component of initiation of parturition in humans?

A. Increase in oestrogen and progesterone ratio

B. Synthesis of prostaglandins

C. Release of oxytocin

D. Release of prolactin

 

Q. 161 Which of the following viruses is not transferred through semen of an infected male?

A. Hepatitis B virus

B. Human immunodeficiency virus

C. Chikungunya virus

D. Ebola virus

 

Q. 162 Which of the following cells during gametogenesis is normally diploid?

A. Primary polar body

B. Spermatid

C. Spermatogonia

D. Secondary polar body

 

Q. 163 Hysterectomy is surgical removal of

A. Uterus

B. Prostate gland

C. Vas deferens

D. Mammary glands

 

Q. 164 Which of the following is not sexually transmitted disease?

A. Syphilis

B. Acquired immune deficiency syndrome (AIDS)

C. Trichomoniasis

D. Encephalitis

 

Q. 165 An abnormal human baby with ‘XXX’ sex chromosomes was born due to

A. Formation of abnormal sperms in the father

B. Formation of abnormal ova in the mother

C. Fusion of two ova and one sperm

D. Fusion of two sperms and one ovum

 

Q. 166 Alleles are

A. Different phenotype

B. True breeding homozygotes

C. Different molecular forms of a gene

D. Heterozygotes

 

Q. 167 A man with blood group ‘A’ marries a woman with blood group ‘B’. What are all the possible blood groups of their offspring?

A. A and B only

B. A, B and AB only

C. A, B, AB and O

D. O only

 

Q. 168 Gene regulation governing lactose operon of E. coli that involves the lac I gene product is

A. Positive and inducible because it can be induced by lactose

B. Negative and inducible because repressor protein prevents transcription

C. Negative and repressible because repressor protein prevents transcription

D. Feedback inhibition because excess of β – galactosidase can switch off transcription

 

Q. 169 In sea urchin DNA, which is double stranded, 17% of the bases were shown to be cytosine. The percentages of the other three bases expected to be present in the DNA are

A. G 34%, A 24.5%, T 24.5%

B. G 17%, A 16.5%, T 32.5%

C. G 17%, A 33%, T 33%

D. G 8.5%, A 50%, T 24.5%

 

Q. 170 Which of the following had the smallest brain capacity?

A. Homo erectus

B. Homo sapiens

C. Homo neanderthalensis

D. Homo habilis

 

Q. 171 A population will not exist in Hardy–Weinberg equilibrium is

A. Individuals mate selectively

B. There are no mutations

C. There is no migration

D. The population is large

 

Q. 172 Match each disease with its correct type of vaccine for (a), (b), (c) and (d) are:

A. (ii) (i) (iii) (iv)

B. (iii) (ii) (iv) (i)

C. (iv) (iii) (ii) (i)

D. (i) (ii) (iv) (iii)

 

Q. 173 HIV that causes AIDS, first starts destroying

A. B-lymphocytes

B. Leukocytes

C. Helper T-lymphocytes

D. Thrombocytes

 

Q. 174 To active form of Entamoeba histolytica feeds upon

A. Erythrocytes mucosa and submucosa of colon

B. Mucosa and submucosa of colon only

C. Food n intestine

D. Blood only

 

Q. 175 High value of BOD (biochemical oxygen demand) indicates that

A. Water is pure

B. Water is highly polluted

C. Water is less polluted

D. Consumption of organic matter in the water is higher by the microbes

 

Q. 176 Most animals are tree dwellers in a

A. Coniferous forest

B. Thorn woodland

C. Temperate deciduous forest

D. Tropical rainforest

 

Q. 177 The following graph depicts changes in two populations (A and B) of herbivores in a grassy field. A possible reason for these changes is that

A. Both plant populations in this habitat decreased

B. Population B compete more successfully for food than population A

C. Population A produced more offspring than population B

D. Population A consumed the members of population B

 

Q. 178 Cryopreservation of gametes of threatened species in viable and fertile condition can be referred to as

A. In situ conservation of biodiversity

B. Advanced ex situ conservation of biodiversity

C. In situ conservation by sacred groves

D. In situ cryo-conservation of biodiversity

 

Q. 179 Rachel Carson’s famous book ‘Silent spring’ is related to

A. Pesticide pollution

B. Noise pollution

C. Population explosion

D. Ecosystem management

 

Q. 180 Which of the following is not one of the prime health risks associated with greater UV radiation through the atmosphere due to depletion of stratospheric ozone?

A. Increased skin cancer

B. Reduced immune system

C. Damage to eyes

D. Increased liver cancer

 

 

Answer Sheet
Question 1 2 3 4 5 6 7 8 9 10
Answer C B B B C C A A C D
Question 11 12 13 14 15 16 17 18 19 20
Answer C D B C B C D C C C
Question 21 22 23 24 25 26 27 28 29 30
Answer B C C B C D C A A B
Question 31 32 33 34 35 36 37 38 39 40
Answer C D D A B C A A B B
Question 41 42 43 44 45 46 47 48 49 50
Answer B B B D C B C C C A
Question 51 52 53 54 55 56 57 58 59 60
Answer C C D B D C A A A C
Question 61 62 63 64 65 66 67 68 69 70
Answer B B B B C A A D B D
Question 71 72 73 74 75 76 77 78 79 80
Answer A D D C D A C C A A
Question 81 82 83 84 85 86 87 88 89 90
Answer A A C C A D B A B C
Question 91 92 93 94 95 96 97 98 99 100
Answer B C D D C C B C C B
Question 101 102 103 104 105 106 107 108 109 110
Answer D A B B B A D B A D
Question 111 112 113 114 115 116 117 118 119 120
Answer B A B A A D B A B D
Question 121 122 123 124 125 126 127 128 129 130
Answer D B C C C A D B B B
Question 131 132 133 134 135 136 137 138 139 140
Answer B A B D B C C B C D
Question 141 142 143 144 145 146 147 148 149 150
Answer C D C D B C A C B C
Question 151 152 153 154 155 156 157 158 159 160
Answer D A C D B B C C A D
Question 161 162 163 164 165 166 167 168 169 170
Answer C C A D B C C B C D
Question 171 172 173 174 175 176 177 178 179 180
Answer A C C A B D B B A D

NEET 2014 Previous Year Paper

NEET 2014

Section

Questions

Marks

Physics

45 Questions (1 – 45)

180

Chemistry

45 Questions (46 – 90)

180

Biology

90 Questions (91 – 180)

360

Q. 1 If force (F), velocity (V) and time (T) are take As fundamental units, then the dimensions of mass are :

A. [F V F⁻¹]

B. [F V T⁻²]

C. [F V⁻¹ T⁻¹]

D. [F V⁻¹ T]

 

Q. 2 A projectile is fired from the surface of the earth with a velocity of 5ms⁻¹ and angle θ with the horizontal. Another projectile fired another planet with a velocity of 3 ms⁻¹ at the same angle follows a trajectory which is identical with the trajectory of the projectile fired from the earth. The value of the acceleration due to gravity on the planet is (in ms⁻²) is: (given g = 9.8 ms⁻²)

A. 3.5

B. 5.9

C. 16.3

D. 110.8

 

Q. 3 A particle is moving such that its position coordinates (x,y)are (2 m, 3 m) at time t = 0, (6 m, 7 m) at time t = 2 s and (13 m, 14 m) at time t = 5s

Average velocity vector Vav from t = 0 to t = 5 s is: 

A. 1/5 * (13 i ̂ +14 i ̂ )

B. 7/3 * (i ̂ + i ̂)

C. 2 * (i ̂ + i ̂)

D. 11/5 * (i ̂ + i ̂)

 

Q. 4 A system consists of three masses m1, m2 and m3 connected by a strings passing over a pulley P. The mass m1 hangs freely and m2 and m3 are on a rough horizontal table (the coefficient of friction = u) the pulley if frictionless and of negligible mass. The downward acceleration of mass m1 is :

A. g(1-gμ)/9

B. 2gμ/3

C. g(1-2μ)/3

D. g(1-2μ)/2

 

Q. 5 The force ‘F’ acting on a particle of mass ‘m’ is indicated by the force — time graph shown below. The change in momentum of the particle over the time interval from zero

A. 24 Ns

B. 20 Ns

C. 12 Ns

D. 6 Ns

 

Q. 6 A balloon with mass ‘m’ descends down with an acceleration ‘a’ (where a < g). How much mass should be removed from it so that it starts moving up with an acceleration ‘a’?

A. 2ma/(g+a)

B. 2ma/(g-a)

C. ma/(g+a)

D. ma/(g-a)

 

Q. 7 A body of mass (4m) is laying in x-y plane at rest. It suddenly explodes into three pieces. Two pieces each of mass (m) move perpendicular to each other with equal speeds (v). The total kinetic energy generated due to explosion is :

A. Mv²

B. 3/2 mv²

C. 2 mv²

D. 4 mv²

 

Q. 8 The oscillation of a body on a smooth horizontal surface is represented by the  equation, Where, X = A cos (ωt); X = displacement at time t; ω = frequency of oscillation. Which one of the following graphs shows correctly the variation ‘a’ with ‘t’? Here a = acceleration at time t 

T = time period

A. (1)

B. (2)

C. (3)

D. (4)

 

Q. 9 A solid cylinder of mass 50 g and radius 0.5 m is free to rotate about the horizontal axis. A massless string is wound round the cylinder with one end attached to it and other hanging freely. Tension in the string required to produce an angular acceleration of 2 revolutions s⁻² is :

A. 25 N

B. 50 N

C. 78.5 N

D. 157 N

 

Q. 10 The ratio of the accelerations for a solid sphere (mass ‘m’ and radius ‘R’ rolling down an incline of angle ‘θ’ Without slipping and slipping down the incline with rolling is :

A. 5:7

B. 2:3

C. 2:5

D. 7:5

 

Q. 11 A black hole is an object whose gravitational field is so strong that even light cannot escape from it. To what approximate radius would earth (mass = 5.98 x 10²⁴ kg) have to be compressed to be a black hole ?

A. 10⁻⁹ m

B. 10⁻⁶ m

C. 10⁻² m

D. 100 m

 

Q. 12 Dependence of intensity of gravitational field (E) of earth with distance (r) from centre of earth is correctly represented by :

A. (1)

B. (2)

C. (3)

D. (4)

 

Q. 13 Copper of fixed volume ‘V’ is drawn into wire of length ‘l’. When the wire is subjected to a constant force ‘F’ the extension produced in the wire is ‘Δl’. Which of the following graphs is a straight line?

A. Δl versus 1/l

B. ΔI versus l²

C. ΔI versus 1/l²

D. ΔI versus l

 

Q. 14 A certain number of spherical drops of liquid of radius ‘r’ coalesce to form a single drop of radius ‘R’ and volume ‘V’. If ‘T’ is the surface tension of the liquid, then :

A. Energy = 4VT (1/e – 1/R) is released.

B. Energy = 3VT (1/r + 1/R) is absorbed

C. Energy = 3VT (1/r – 1/R) is released

D. Energy is neither released nor absorbed

 

Q. 15 Steam at 100°C is passed into 20 g of water at 10°C. When water acquires a temperature of 80°C the mass of water present will be :

[Take specific heat of water = 1 cal g⁻¹ °C⁻¹ and latent heat of steam = 540 cal g⁻¹]

A. 24 g

B. 31.5 g

C. 42.5 g

D. 22.5 g

 

Q. 16 Certain quantity of water cools from 70°C to 60°C in the first 5 minutes and to 54°C in the next 5 minutes. The temperature of the surroundings is :

A. 45°C

B. 20°C

C. 42°C

D. 10°C

 

Q. 17 A monatomic gas at pressure P, having a volume V expands isothermally to a volume 2V and then adiabatically to a volume 16 V. The final pressure if the gas is : (take y: 5/3)

A. 64 P

B. 32 Po

C. P/ 64

D. 16 P

 

Q. 18 A thermodynamic system undergoes cyclic process ABCDA as shown in Fig. The work done by the system in the cycle is :

A. P0V0

B. 2 P0V0

C. P0V0/2

D. Zero

 

Q. 19 The mean free path of molecules of a gas, (radius is inversely proportional to :

A. r³

B. r²

C. r

D. √r

 

Q. 20 If n₁,n₂ and n₃ are the fundamental frequencies three segments into which a string is divided, the original fundamental frequency n of the string is given by :

A. 1/n = 1/n₁ + 1/n₂ + 1/n₃

B. 1/sqrt(n) = 1/sqrt(n₁) + 1/sqrt(n₂) + 1/sqrt(n₃)

C. sqrt(n) = sqrt(n₁) + sqrt(n₂) + sqrt(n₃)

D. n = n₁ + n₂ + n₃

 

Q. 21 The number of possible natural oscillations of column in a pipe closed at one end of length 85 cm whose frequencies lie below 1250 Hz are :

(velocity of sound = 340 ms⁻¹)

A. 4

B. 5

C. 7

D. 6

 

Q. 22 A speeding motorcyclist sees traffic jam ahead him. He slows down to 36 km/hour. He finds the traffic has eased and a car moving ahead of him 18 km/hour is honking at a frequency of 1392 Hz. The speed of sound is 343 m/s, the frequency of the honk as heard by him will be :

A. 1332 Hz

B. 1372 Hz

C. 1412 Hz

D. 1454 Hz

 

Q. 23  Two thin dielectric slabs of dielectric constants K1 and K2 (K1

A. (1)

B. (2)

C. (3)

D. (4)

 

Q. 24 A conducting sphere of radius R is given a charge Q. The electric potential and the electric field at the centre of the sphere respectively are

A. Zero and Q/(4 π ε R^2)

B. Q/(4 π ε R) and Zero

C. Q/(4 π ε R) and Q/(4 π ε R^2)

D. Both are Zero

 

Q. 25 In a region the potential is represented by V (x,y,z) = 6x – 8xy – 8y + 6yz, where v is in volts and x, y, z are in meters. The electric force experienced by a change of 2 coulomb situated at point (1,1,1) is :

A. 6√5 N

B. 30 N

C. 24 N

D. 4√35 N

 

Q. 26 Two cities are 150 km apart. Electric power is sent from one city to another city through copper wires. The fall of potential per km is 8 volt and the average resistance per km is 0.5 Ω. The power loss in the wire is :

A. 19.2 w

B. 19.2 kW

C. 19.2 J

D. 12.2 kW

 

Q. 27 The resistances in the two arms of the meter bridge are 5 Ω and R Ω respectively. When the resistance R is shunted with an equal resistance; the new balance point is at 1.6l₁. The resistance ‘R’ is:

A. 10 Ω

B. 15 Ω

C. 20 Ω

D. 25 Ω

 

Q. 28 A potentiometer circuit has been set up for finding the internal resistance of given cell. The main battery, used across the potentiometer wire, has an emf of 2.0 V and a negligible internal resistance. The potentiometer wire itself is 4 m long. When the resistance R connected across the given cell, has values of.

(1) Infinity

(2) 9.5 Ω

The ‘balancing lengths’ on the potentiometer wire are found to be 3 m and 2.85 m, Respectively. The value of internal resistance of the cell is:

A. 0.25 Ω

B. 0.95 Ω

C. 0.5 Ω

D. 0.75 Ω

 

Q. 29 Following figures show the arrangement of bar magnets in different configurations. Each magnet has magnetic dipole moment⃗ m . Which configuration has highest net magnetic dipole moment?

A. (a)

B. (b)

C. (c)

D. (d)

 

Q. 30 In an ammeter 0.2% of main current passes through the galvanometer. If resistance of galvanometer is G, the resistance of ammeter will be:

A. (1/499) * G

B. (499/500) * G

C. (1/500) * G

D. (500/499) * G

 

Q. 31 Two identical long conducting wires AOB and COD are placed at right angle to each other, with one above other such that ‘O’ is their common point for the two. The wires carry I1 and I2 currents, respectively. Point ‘P’ is lying at distance‘d’ from ‘O’ along a direction perpendicular to the plane containing the wires. The magnetic field at the point ‘P’ will be: 

A. μ/2πd * (I1 / I2)

B. μ/2πd * (I1 + I2)

C. μ/2πd * (I1^2 – I2^2)

D. μ/2πd * sqrt (I1^2 + I2^2)

 

Q. 32 A thin semicircular conducting ring (PQR) of radius ‘r’ is falling with its plane vertical in a horizontal magnetic field B, as shown in figure. The potential difference developed across the ring when its speed is v, is:

A. Zero

B. Bvπr²/2 and P is at higher potential

C. πrBv and R is at higher potential

D. 2rBv and R is at higher potential

 

Q. 33 A transformer having efficiency of 90% is working on 200 V and 3 kW power supply. If the current in the secondary coil is 6A, the voltage across the secondary coil and the current in the primary coil respectively are :

A. 300 V, 15 A

B. 450 V, 15 A

C. 450 V, 13.5 A

D. 600 V, 15 A

 

Q. 34 Light with an energy flux of 25 x 10⁴ Wm⁻² falls on a perfectly reflecting surface at normal incidence. If the surface area is 15 cm², the average force exerted on the surface is :

A. 1.25 x 10⁻⁶ N

B. 2.50 x 10⁻⁶ N

C. 1.20 x 10⁻⁶ N

D. 3.0 x 10⁻⁶ N

 

Q. 35 A beam of light of λ = 600 nm from a distant source falls on a single slit 1 mm wide and the resulting diffraction pattern is observed on a screen 2 m away. The distance between first dark fringes on either side of the central bright fringe is :

A. 1.2 cm

B. 1.2 mm

C. 2.4 cm

D. 2.4 mm

 

Q. 36 In the Young’s double — slit experiment, the intensity of light at a point on the screen where the path difference is λ is K, (λ being the wavelength of light used). The intensity at point where the path difference is λ/4 will be:

A. K

B. K/4

C. K/2

D. Zero

 

Q. 37 If the focal length of objective lens is increased then magnifying power of:

A. Microscope will increase but that of telescope decrease.

B. Microscope and telescope both will increase

C. Microscope and telescope both will decrease

D. Microscope will decrease but that of telescope will increase.

 

Q. 38 The angle of prism is ‘A’ One of its refracting surfaces is silvered. Light rays falling at an angle of incidence 2A on the first surface returns back through the same path after suffering reflection at the silvered surface. The refractive index μ of the prism is

A. 2 sin A

B. 2 cos A

C. 1/2 cos A

D. Tan A

 

Q. 39 When the energy of the incident radiation is increased by 20% the kinetic energy of the photoelectrons emitted from a metal surface increased from 0.5 eV to 0.8 eV. The work function of the metal is

A. 0.65 eV

B. 1.0 eV

C. 1.3 eV

D. 1.5 eV

 

Q. 40 If the kinetic energy of the particle is increased to 16 times its previous value, the percentage change in the de-Broglie wavelength of the particle is :

A. 25

B. 75

C. 60

D. 50

 

Q. 41 Hydrogen atom in ground state is excited by a monochromatic radiation of λ = 975 A. Number of spectral lines in the resulting spectrum emitted will be :

A. 3

B. 2

C. 6

D. 10

 

Q. 42 The Binding energy per nucleon of ⁷₃Li and ⁴₂He nuclei are 5.60 MeV and 7.06 MeV respectively. In the nuclear reaction ⁷₃Li + ¹₁H → ⁴₂He + ⁴₂He +Q, the value of energy Q released is :

A. 19.6 MeV

B. -2.4 MeV

C. 8.4 MeV

D. 17.3 MeV

 

Q. 43 A radio isotope ‘X’ with a half life 1.4 x 10⁹ years decays to ‘Y’ which is stable. A sample of the rock from a cave was found to contain ‘X’ and ‘Y’ in the ratio 1:7. The age of the rock is : 

A. 1.96 x 10⁹ years

B. 3.92 x 10⁹ years

C. 4.20 x 10⁹ years

D. 8.40 x 10⁹ years

 

Q. 44 The given graph represents V — I characteristics for semiconductor device.

Which of the following statement is correct?

A. It is V — I characteristics for solar cell where point A represents open circuit voltage and point b short circuit current.

B. It is for s solar cell and points A and B represent open circuit voltage and current, respectively.

C. It is for a photodiode and points A and B represent open circuit voltage and current, respectively

D. It is for LED and points A and B represent open circuit voltage and short circuit current, respectively

 

Q. 45 The barrier potential of a p – n junction depends on :

(a) Type of semiconductor material

(b) Amount of doping

(c) Temperature

Which one of the following is correct?

A. (a) and (b) only

B. (c) only

C. (b) and (c) only

D. (a), (b) and (c)

 

Q. 46 What is the maximum number of orbitals that can be identified with the following quantum numbers?

n=3, l=1, m₁=0

A. 1

B. 2

C. 3

D. 4

 

Q. 47 Calculate the energy in joule corresponding to light of wavelength 45 nm :

(Planck’s constant h = 6.63 x 10⁻³⁴ Js ; speed of light c = 3 x 10⁸ ms⁻¹)

A. 6.67 x 10¹⁵

B. 6.67 x 10¹¹

C. 4.42 x 10⁻¹⁵

D. 4.42 x 10⁻¹⁸

 

Q. 48 Equal masses of H₂, 0₂ and methane have been taken in a container of volume V at temperature 27° C in identical conditions. The ratio of the volumes of gases H₂ : O₂ : methane would be :

A. 8 : 16 : 1

B. 16 : 8 : 1

C. 16: 1 : 2

D. 8 : 1 : 2

 

Q. 49 If a is the length of the side of a cube, the distance between the body centered atom and one corner atom in the cube will be

A. 2/√3 a

B. 4/√3 a

C. √3/4 a

D. √3/2 a

 

Q. 50 Which property of colloids is not dependent on the charge on colloidal particles ?

A. Coagulation

B. Electrophoresis

C. Electro — osmosis

D. Tyndall effect

 

Q. 51 Which of the following salts will give highest pH in water ?

A. KCl

B. NaCl

C. Na₂CO₃

D. CuSO₄

 

Q. 52 Of the following 0.10 aqueous solutions, which one will exhibit the largest freezing point depression?

A. KCl

B. C₆H₁₂O₆

C. Al₂(SO₄)₃

D. K₂SO₄

 

Q. 53 When 22.4 liters of H₂ (g) is mixed with 11.2 liters of Cl₂ (g), each at S.T.P, the moles of HCl (g) formed is equal to :

A. 1 mol of HCl (g)

B. 2 mol of HCl (g)

C. 0.5 mol of HCl (g)

D. 1.5 mol of HCl (g)

 

Q. 54 When 0.1 mol MnO²₄⁻ is oxidized the quantity of electricity required to completely oxidize MnO ²₄⁻ to MnO⁻₄ is :

A. 96500 c

B. 2 x 96500 c

C. 9650 c

D. 96.50 c

 

Q. 55 Using the Gibbs energy change, ΔG° = +63.3 kJ, for the following reaction,

Ag₂CO₃ (s) ↔ 2Ag⁺ (aq) + CO₃²⁻ (aq)

The Ksp of Ag₂CO₃(s) in water at 25°C is:

(R = 8.314 J K⁻¹ mol⁻¹)

A. 3.2 x 10⁻²⁶

B. 8.0 x 10⁻¹²

C. 2.9 x 10⁻³

D. 7.9 x 10⁻²

 

Q. 56 The weight of silver (at.wt = 108) displaced by a quantity of electricity which displaces 5600 mL of 0₂ at STP will be :

A. 5.4 g

B. 10.8 g

C. 54.0 g

D. 108.0 g

 

Q. 57 Which of the following statements is correct for the spontaneous adsorption of a gas?

A. S is negative and, therefore, H should be highly positive.

B. S is negative and therefore, H should be highly negative.

C. S is positive and therefore, H should be negative.

D. S is positive and, therefore, H should also be highly positive.

 

Q. 58 For the reversible reaction :

N₂ (g) + 3H₂(g) ↔ 2NH₃ (g) + heat

The equilibrium shifts in forward direction :

A. By increasing the concentration of NH₃ (g)

B. By decreasing the pressure

C. By decreasing the concentration of N₂ (g) and H₂ (g)

D. By increasing pressure and decreasing temperature

 

Q. 59 For the reaction :

X₂O₄ → 2X0₂ (g)

U = 2.1 k cal, S = 20 cal K⁻¹ at 300 K

Hence, G is

A. 2.7 k cal

B. -2.7 k cal

C. 9.3 k cal

D. -9.3 k cal

 

Q. 60 For a given exothermic reaction, Kp and K’p are the equilibrium constants at temperatures T₁ and T₂, respectively. Assuming that heat of reaction is constant in temperature range between T₁ and T₂, it is readily observed that :

A. Kp > K’p

B. Kp < K’p

C. KP =K’p

D. Kp = 1/K’p

 

Q. 61 Which of the following orders of ionic radii is correctly represented ?

A. H⁻ > H⁺ > H

B. Na⁺ > F⁻ > 0²⁻

C. F⁻ > 0²⁻ > Na⁺

D. Al³⁺ > Mg²⁺ > N³⁻

 

Q. 62 1.0 g of magnesium is burnt with 0.56 g O₂ in a closed vessel. Which reactant is left in excess and how much?

(At.wt.Mg = 24; O = 16)

A. Mg, 0.16 g

B. O₂ , 0.16 g

C. Mg, 0.44 g

D. O₂, 0.28 g

 

Q. 63 The pair of compounds that can exist together is :

A. FeCl₃, SnCl₂

B. HgCl₂, SnCl₂

C. FeCl₂, SnCl₂

D. FeCl₃, KI

 

Q. 64 Be²⁺ is isoelectronic with which of the following ions?

A. H⁺

B. Li⁺

C. Na⁺

D. Mg²⁺

 

Q. 65 Which of the following molecules has the maximum dipole moment ?

A. CO₂

B. CH₄

C. NH₃

D. NF₃

 

Q. 66 Which one of the following species has plane triangular shape?

A. N₃

B. NO₃⁻

C. NO₂⁻

D. CO₂

 

Q. 67 Acidity of diprotic acids in aqueous solutions increase in the order

A. H₂S < H₂Se < H₂Te

B. H₂Se < H₂S > H₂Te

C. H₂Te < H₂S < H₂Se

D. H₂Se < H₂Te < H₂S

 

Q. 68 (a) H₂O₂ + O3 → H₂O + 2O₂

(b) H₂O₂ + Ag₂O → 2Ag + H₂O + O₂

Role of hydrogen peroxide in the above reactions is respectively:

A. Oxidizing in (a) and reducing in (b)

B. Reducing in (a) and oxidizing in (b)

C. Reducing in (a) and (b)

D. Oxidizing in (a) and (b)

 

Q. 69 Artificial sweetener which is stable under cold conditions only is :

A. Saccharine

B. Sucralose

C. Aspartame

D. Alitame

 

Q. 70 In acidic medium, H₂O₂ changes Cr₂O₇⁻² to CrO₅ which has two (-O-O-) bonds. Oxidation state of Cr in CrO₅ is :

A. +5

B. +3

C. +6

D. -10

 

Q. 71 The reaction of aqueous KMnO₄ with H₂O₂ in acidic conditions gives :

A. Mn⁴⁺ and O₂

B. Mn²⁺ and O₂

C. Mn²⁺ and O₃

D. Mn⁴⁺ and MnO₂

 

Q. 72 Among the following complexes the one which shows zero crystal field stabilization energy (CFSE) is :

A. [Mn(H₂O)₆]³⁺

B. [Fe(H₂O)₆]³⁺

C. [Co(H₂O)₆]²⁺

D. [Co(H₂O)₆]³⁺

 

Q. 73 Magnetic moment 2.83 BM is given by which of the following ions?

(At.nos. Ti = 22, Cr = 24, Mn = 25, Ni = 28)

A. Ti³⁺

B. Ni²⁺

C. Cr³⁺

D. Mn²⁺

 

Q. 74 Which of the following complexes is used to be as anticancer agent ?

A. mer — [Co(NH₃)₃Cl]

B. cis — [PtCl₂(NH₃)₂]

C. cis — K₂[PtCl₂Br₂]

D. Na₂COCl₄

 

Q. 75 Reasons of lanthanoid contraction is :

A. Negligible screening effect of ‘f’ orbitals

B. Increasing Nuclear charge

C. Decreasing nuclear charge

D. Decreasing screening effect

 

Q. 76 In the following reaction, the product (A)

A. (1)

B. (2)

C. (3)

D. (4)

 

Q. 77 Which of the following will be most stable diazonium salt RN₂⁺X⁻ ?

A. CH₃N₂⁺X⁻

B. C₆H₅N₂⁺X⁻

C. CH₃CH₂N₂⁺X⁻

D. C₆H₅CH₂N₂⁺X⁻

 

Q. 78 D (+) glucose reacts with hydroxyl amine and yields an oxime. The structure of the oxime would be :

 

A. a

B. b

C. c

D. d

 

Q. 79 Which of the following hormones is produced under the condition of stress which stimulates glycogenolysis in the liver of human beings ?

A. Thyroxine

B. Insulin

C. Adrenaline

D. Estradiol

 

Q. 80 Which one of the following is an example of a thermosetting polymer?

A. (1)

B. (2)

C. (3)

D. (4)

 

Q. 81 Which of the following organic compounds polymerizes to form the polyester Dacron?

A. Propylene and para HO — (C₆H₄) — OH

B. Benzoic acid and ethanol

C. Terephthalic acid and ethylene glycol

D. Benzoic acid and para HO —(C₆H₄) — OH

 

Q. 82 Which one of the following is not a common component of photochemical smog ?

A. Ozone

B. Acrolein

C. Peroxyacetyl nitrate

D. Chlorofluorocarbons

 

Q. 83 In the Kjeldahl’s method for estimation of nitrogen present in a soil sample, ammonia evolved from 0.75 g sample neutralized 10 mL of 1M H₂SO₄. The percentage of nitrogen in the soil is :

A. 37.33

B. 45.33

C. 35.33

D. 43.33

 

Q. 84 What products are formed when the following compound is treated with Br in the presence of FeBr₃ ?

A. (1)

B. (2)

C. (3)

D. (4)

 

Q. 85 Which of the following compounds will undergo racemisation when solution of KOH hydrolyses?

A. (i) and (ii)

B. (ii) and (iv)

C. (iii) and (iv)

D. (i) and (iv)

 

Q. 86 Among the following sets of reactants which one produces anisole?

A. CH₃CHO;RMgX

B. C₆H₅OH;NaOH;CH₃I

C. C₆H₅OH;neutral FeCl₃

D. C₆H₅ — CH₃;CH₃COCl;AlCl₃

 

Q. 87 Which of the following will not be soluble in sodium hydrogen carbonate?

A. 2,4,6 — trinitrophenol

B. Benzoic acid

C. o — Nitrophenol

D. Benzenesulfonic acid

 

Q. 88 Which one is most reactive towards Nucleophilic addition reaction?

A. (1)

B. (2)

C. (3)

D. (4)

 

Q. 89 Identity Z in the sequence of reactions :

CH₃CH₂CH = CH₂ _______ Hbr/H₂O₂ → Y → C₃H₅ONaZ

A. CH₃ — (CH₂)₃ — O — CH₂CH₃

B. (CH₃)2CH₂ — O — CH₂CH₃

C. CH₃(CH₂)₄ — O — CH₃

D. CH₃CH₂ — CH (CH₃) — O — CH₂CH₃

 

Q. 90 Which of the following organic compounds has same hybridization as its combustion product — (CO₂)?

A. Ethane

B. Ethyne

C. Ethene

D. Ethanol

 

Q. 91 Which one of the following shows isogamy with non — flagellated garnets ?

A. Sargassum

B. Ectocarpus

C. Ulothrix

D. Spirogyra

 

Q. 92 Five kingdom system of classification suggested by R.H. Whittaker is not based on :

A. Presence or absence of a well defined nucleus

B. Mode of reproduction

C. Mode of nutrition

D. Complexity of body organization

 

Q. 93 Which one of the following fungi contains hallucinogens ?

A. Morchella esculenta

B. Amanita muscaria

C. Neurospora sp.

D. Ustilago sp.

 

Q. 94 Archaebacteria differ from eubacteria in :

A. Cell membrane structure

B. Mode of nutrition

C. Cell shape

D. Mode of reproduction

 

Q. 95 Which one of the following is wrong about Chara?

A. Upper oogonium and lower round antheridium

B. Globule and nucule present on the same plant

C. Upper antheridium and lower oogonium

D. Globule is male reproductive structure

 

Q. 96 Which of the following is responsible for peat formation ?

A. Marchantia

B. Riccia

C. Funaria

D. Sphagnum

 

Q. 97 Placenta and pericarp are both edible portions in:

A. Apple

B. Banana

C. Tomato

D. Potato

 

Q. 98 When the margins of sepals or petals overlap one another without any particular direction, the condition is termed as :

A. Vexillary

B. Imbricate

C. Twisted

D. Valvate

 

Q. 99 You are given a fairly old piece of dicot stem and a dicot root. Which of the following anatomical structures will you use to distinguish between the two?

A. Secondary xylem

B. Secondary phloem

C. Protoxylem

D. Cortical cells

 

Q. 100 Which one of the following statement is correct ?

A. The seed in grasses is not endospermic

B. Mango is parthenocarpic fruit

C. A proteinaceous aleurone layer is present in maize grain

D. A sterile pistil is called a stainamode

 

Q. 101 Tracheids differ from other tracheary elements in :

A. Having casparian strips

B. Being imperforate

C. Lacking nucleus

D. Being lignified

 

Q. 102 An example of edible underground stem is :

A. Carrot

B. Groundnut

C. Sweet potato

D. Potato

 

Q. 103 Which structure performs the function of mitochondria in bacteria?

A. Nucleoid

B. Ribosomes

C. Cell wall

D. Mesosomes

 

Q. 104 The solid linear cytoskeletal elements having a diameter of 6 nm and made up of a single type of monomer are known as :

A. Microtubules

B. Microfilaments

C. Intermediate filaments

D. Lamins

 

Q. 105 The osmotic expansion of a cell kept in water is chiefly regulated by:

A. Mitochondria

B. Vacuoles

C. Plastics

D. Ribosomes

 

Q. 106 During which phase (s) of cell cycle, amount of DNA in a cell remains at 4C level if the initial amount is denoted as 2C?

A. G₀ and G₁

B. G₁ and S

C. Only G₂

D. G₂ and M

 

Q. 107 Match the following and select the correct answer :

(a) Centriole (i) Infoldings in mitochondria

(b) Chlorophyll (ii) Thylakoids

(c) Cristae (iii) Nucleic acids

(d) Ribozymes (iv) basal body cilia or flagella

A. (1)

B. (2)

C. (3)

D. (4)

 

Q. 108 Dr. F went noted that if coleptile were removed and placed on agar for one hour, the agar would produce a bending when placed on one side of freshly-cut-coleoptile stumps. Of what significance is this experiment ?

A. It made possible the isolation and exact identification of auxin

B. It is the basis for quantitative determination of small amounts of growth — promoting substances

C. It supports the hypothesis that IAA is auxin

D. It demonstrated polar movement of auxins

 

Q. 109 Deficiency symptoms of nitrogen and potassium are visible first in :

A. Senescent leaves

B. Young leaves

C. Roots

D. Buds

 

Q. 110 In which one of the following processes CO₂ is not released ?

A. Aerobic respiration in plants

B. Aerobic respiration in animals

C. Alcoholic fermentation

D. Lactate fermentation

 

Q. 111 Anoxygenic photosynthesis is characteristics of :

A. Rhodospirillum

B. Spirogyra

C. Chlamydomonas

D. Ulva

 

Q. 112 A few normal seedlings of tomato were kept in a dark room. After a few days they were found to have become while — colored like albinos. Which of the following terms will you use to describe them?

A. Mutated

B. Embolized

C. Etiolated

D. Defoliated

 

Q. 113 Which one of the following growth regulators is known as ‘ stress hormone’?

A. Abscisic acid

B. Ethylene

C. GA₃

D. Indole acetic acid

 

Q. 114 Geitonogamy involves

A. Fertilization of a flower by the pollen from another flower of the same plant

B. Fertilization of a flower by the pollen from the same flower

C. Fertilization of a flower by the pollen from a flower of another plant in the same population

D. Fertilization of a flower by the pollen from a flower of another plant belonging to a distant population

 

Q. 115 Male gametophyte with least number of cells is present in:

A. Pteris

B. Funaria

C. Lilium

D. Pinus

 

Q. 116 An aggregate fruit is one which develops from:

A. Multicarpellary syncarpous gynoecium

B. Multicarpellary apocarpous gynoecium

C. Complete inflorescence

D. Multicarpellary superior ovary

 

Q. 117 Pollen tablets are available in the market for:

A. In vitro fertilization

B. Breeding programs

C. Supplementing food

D. Ex situ conservation

 

Q. 118 Function of filiform apparatus is to

A. Recognize the suitable pollen at stigma

B. Stimulate division of generative cell

C. Produce nectar

D. Guide the entry of pollen tube

 

Q. 119 Non — albuminous seed is produced in :

A. Maize

B. Castor

C. Wheat

D. Pea

 

Q. 120 Which of the following shows coiled RNA strand and capsomeres ?

A. Poliovirus

B. Tobacco mosaic virus

C. Measles virus

D. Retrovirus

 

Q. 121 Which one of the following is wrongly matched ?

A. Transcription — Writing information From DNA to t — RNA

B. Translation — Using information in m — RNA to make protein

C. Repressor protein — Binds to operator to stop enzyme synthesis

D. Operon – Structural genes, operator and promoter

 

Q. 122 Transformation was discovered by :

A. Meselson and stahl

B. Hershey and chase

C. Griffith

D. Watson and Crick

 

Q. 123 Fruit color in squash is an example of:

A. Recessive epistasis

B. Dominant epistasis

C. Complementary genes

D. Inhibitory genes

 

Q. 124 Viruses here :

A. DNA enclosed in a protein coat

B. Prokaryotic nucleus

C. Single chromosome

D. Both DNA and RNA

 

Q. 125 The first human hormone produced by recombinant DNA technology is :

A. Insulin

B. Estrogen

C. Thyroxine

D. Progesterone

 

Q. 126 An analysis of chromosomal DNA using the southern hybridization technique does not use:

A. Electrophoresis

B. Blotting

C. Autoradiography

D. PCR

 

Q. 127 In vitro propagation in plants is characterized by :

A. PCR and RAPD

B. Northern blotting

C. Electrophoresis and HPLC

D. Microscopy

 

Q. 128 An alga which can be employed as food for human being is :

A. Ulothrix

B. Chlorella

C. Spirogyra

D. Polysiphonia

 

Q. 129 Which vector can clone only small fragment of DNA ?

A. Bacterial artificial chromosome

B. Yeast artificial chromosome

C. Plasmid

D. Cosmid

 

Q. 130 An example of ex situ conservation is :

A. National park

B. Seed Bank

C. Wildlife Sanctuary

D. Sacred Grove

 

Q. 131 A location with luxuriant growth of lichens on the trees indicates that the

A. Trees are very healthy

B. Trees are heavily infested

C. Location is highly polluted

D. Location is not polluted

 

Q. 132 Match the following and select the correct option :

(a) Earthworm (i) Pioneer species

(b) Succession (ii) Detritivore

(c) Ecosystem service (iii) Natality

(d) Population growth (iv) Pollination

A. (1) 

B. (2)

C. (3)

D. (4)

 

Q. 133 A species facing extremely high risk of extinction in the immediate future is called:

A. Vulnerable

B. Endemic

C. Critically Endangered

D. Extinct

 

Q. 134 The zone of atmosphere in which the ozone layer is present is called :

A. Ionosphere

B. Mesosphere

C. Stratosphere

D. Troposphere

 

Q. 135 The organization which publishes the Red list of species is :

A. ICFRE

B. IUCN

C. UNEP

D. WWF

 

Q. 136 Select the Taxon mentioned that represents both marine and freshwater species :

A. Echinoderms

B. Ctenophora

C. Cephalochordata

D. Cnidaria

 

Q. 137 Which one of the following living organisms completely lacks of cells wall ?

A. Cyanobacteria

B. Sea — Fan (Gorgonia)

C. Saccharomyces

D. Blue — Green algae

 

Q. 138 Planaria possess high capacity of:

A. Metamorphosis

B. Regeneration

C. Alternation of generation

D. Bioluminescence

 

Q. 139 A marine cartilaginous fish that can produce electric current is :

A. Pristis

B. Trygon

C. Trygon

D. Scoliodon

 

Q. 140 Choose the correctly matched pair :

A. Tendon – Specialized connective tissue

B. Adipose tissue — Dense connective tissue

C. Areolar tissue — Loose connective tissue

D. Cartilage — Loose connective tissue

 

Q. 141 Choose the correctly matched pair :

A. Inner lining of salivary ducts — Ciliated epithelium

B. Moist surface of buccal cavity – Glandular epithelium

C. Tubular parts of nephrons — Cuboidal epithelium

D. Inner surface of bronchioles — squamous epithelium

 

Q. 142 In ’s’ phase of the cell cycle

A. Amount of DNA doubles in each cell

B. Amount of DNA remains same in each cell.

C. Chromosome number is increased

D. Amount of DNA is reduced to half in each cell

 

Q. 143 The motile bacteria are able to move by :

A. Fimbriae

B. Flagella

C. Cilia

D. Pili

 

Q. 144 Select the option which is not correct with respect to enzyme action :

A. Substrate binds with enzyme at its active site

B. Addition of lot of succinate does not reverse the inhibition of succinic dehydrogenase by malonate

C. A non — competitive inhibitor binds the enzyme at a site distinct from that which binds the substrate

D. Malonate is a competitive inhibitor of succinic dehydrogenase

 

Q. 145 Which one of the following is a non — reducing carbohydrate?

A. Maltose

B. Sucrose

C. Lactose

D. Ribose 5 — phosphate

 

Q. 146 The enzyme recombines is required at which stage of meiosis :

A. Pachytene

B. Zygotene

C. Diplotene

D. Diakinesis

 

Q. 147 The initial step in the digestion of milk in human is carried out by ?

A. Lipase

B. Trypsin

C. Rennin

D. Pepsin

 

Q. 148 Fructose is absorbed bed into the blood through mucosa cells of intestine by the process called :

A. Active transport

B. Facilitated transport

C. Simple diffusion

D. Co-transport mechanism

 

Q. 149 Approximately seventy percent of carbon — dioxide absorbed by the blood will be transported to the lungs :

A. As bicarbonate ions

B. In the form of dissolved gas molecules

C. By binding to R.B.C

D. As carbamino — hemoglobin

 

Q. 150 Person with blood group AB is considered as universal recipient because he has :

A. Both A and B antigens on RBC but no antibodies in the plasma

B. Both A and B antibodies in the plasma

C. No antigen on RBC and no antibody in the plasma

D. Both A and antigens in the plasma but no antibodies

 

Q. 151 How do parasympathetic neural signals affect the working of the heart ?

A. Reduce both heart rate and cardiac output

B. Heart rate is increased without affecting the cardiac output.

C. Both heart rate and cardiac output increase

D. Heart rate decreases but cardiac output increases.

 

Q. 152 Which of the following causes an increase in sodium reabsorption in the distal convoluted tubule ?

A. Increase in aldosterone levels

B. Increase in antidiuretic hormone levels

C. Decrease in aldosterone levels

D. Decrease in antidiuretic hormone levels

 

Q. 153 Select the correct matching of the type of the joint with the example in human skeletal system :

A. (1)

B. (2)

C. (3)

D. (4)

 

Q. 154 Stimulation of a muscle fiber by a motor neuron occurs at :

A. The neuromuscular junction

B. The transverse tubules

C. The myofibril

D. The sarcoplasmic reticulum

 

Q. 155 Injury localized to the hypothalamus would most likely disrupt:

A. Short — term memory

B. Co-ordination during locomotion

C. Executive function, such as decision making

D. Regulation of body temperature

 

Q. 156 Which one of the following statements is not correct ?

A. Retinal is the light absorbing portion of visual photopigments

B. In retina the rods have the photopigment rhodopsin while cones have three different photopigments

C. Retinal is a derivative of vitamin C.

D. Rhodopsin is the purplish red protein present in rods only.

 

Q. 157 Identify the hormone with its correct matching of source and function :

A. Oxytocin — posterior pituitary, growth and maintenance of mammary glands

B. Melatonin — pineal gland, regulates the normal rhythm of sleep-wake cycle.

C. Progesterone — corpus — luteum, stimulation of growth and activities of female secondary sex organs

D. Atrial natriuretic factor — ventricular wall increases the blood pressure.

 

Q. 158 Fight — or — flight reactions cause activation of:

A. The parathyroid glands, leading to increased metabolic rate.

B. The kidney, leading to suppression of renin angiotensin — aldosterone pathway

C. The adrenal medulla, leading to increased secretion of epinephrine and norepinephrine

D. The pancreas leading to a reduction in the blood sugar levels

 

Q. 159 The shared terminal duct of the productive and urinary system in the human ale is :

A. Urethra

B. Ureter

C. Vas deferens

D. Vasa efferentia

 

Q. 160 The main function of mammalian corpus luteum is to produce :

A. Estrogen only

B. Progesterone

C. Human chorionic gonadotropin

D. Relaxin only

 

Q. 161 Select the correct option describing gonadotropin activity in a normal pregnant female :

A. High level of FSH and LH stimulates the thickening of endometrium

B. High level of FSH and LH facilitate implantation of the embryo

C. High level of hCG stimulates the synthesis of estrogen and progesterone.

D. High level of hCG stimulates the thickening of endometrium.

 

Q. 162 Tubectomy is a method of sterilization in which :

A. Small party of fallopian tube is removed or tied up

B. Ovaries are removed surgically

C. Small party of vas deferens is removed or tied up.

D. Uterus is removed surgically

 

Q. 163 Which of the following is a hormone releasing intraUterine Device (IUD)

A. Multiload 375

B. LNG — 20

C. Cervical cap

D. Vault

 

Q. 164 Assisted reproductive technology, IVF involves transfer of :

A. Ovum into fallopian tube

B. Zygote into the fallopian tube

C. Zygote into the uterus

D. Embryo with 16 blastomeres into the fallopian tube

 

Q. 165 A man whose father was color blind marries a woman who had a color blind mother and normal father. What percentage of male children of this couple will be color blind ?

A. 25%

B. 0%

C. 50%

D. 75%

 

Q. 166 In a population of 1000 individuals 360 belong to genotype AA, 480 to Aa and the remaining 160 to aa. Based on this date, the frequency of allele a in the population is :

A. 0.4

B. 0.5

C. 0.6

D. 0.7

 

Q. 167 A human female with Turner’s syndrome :

A. Has 45 chromosomes with XO

B. Has one additional X chromosome

C. Exhibits male characters

D. Is able to produce children with normal husband

 

Q. 168 Select the correct option :

A. (1)

B. (2)

C. (3)

D. (4)

 

Q. 169 Commonly used vectors for human genome sequencing are :

A. T — DNA

B. BAC and YAC

C. Expression Vectors

D. T/A Cloning Vectors

 

Q. 170 Forelimbs of cat, lizard used in walking forelimbs of whale used in swimming and forelimbs of bats used in flying are an example of:

A. Analogous organs

B. Adaptive radiation

C. Homologous organs

D. Convergent evolution

 

Q. 171 Which one of the following are analogous structures ?

A. Wings of Bat and wings of Pigeon

B. Gills of Prawn and Lungs of Man

C. Thorns of Bougainvillea and Tendrils of Cucurbita

D. Flippers of Dolphin and Legs of Horse

 

Q. 172 Which is the particular type of drug that is obtained from the plant whose one flowering branch is shown below?

A. Hallucinogen

B. Depressant

C. Stimulant

D. Pain — killer

 

Q. 173 At which stage of HIV infection does one usually show symptoms of AIDS ?

A. Within 15 days of sexual contact with an infected person

B. When the infected retro virus enters host cells

C. When HIV damages large number of helper T—Lymphocytes

D. When the viral DNA is produced by reverse transcriptase

 

Q. 174 To obtain virus—free healthy plants from a diseased one by tissue culture technique, which part/parts of the diseased plant will be taken?

A. Apical meristem only

B. Palisade parenchyma

C. Both apical and axillary meristem

D. Epidermis only

 

Q. 175 What gases are produced in anaerobic sludge digesters ?

A. Methane and CO₂ only

B. Methane, Hydrogen, sulphide and CO₂

C. Methane, Hydrogen sulphide and O₂

D. Hydrogen sulphide and CO₂

 

Q. 176 Just as a person moving from Delhi to Shimla to escape the heat for the duration of hot summer, thousands of migratory birds from Siberia and other extremely cold northern regions move to :

A. Western Ghat

B. Meghalaya

C. Corbett National Park

D. Keoladeo National Park

 

Q. 177 Given below is a simplified model of phosphorus cycling in a terrestrial ecosystem with four blanks (A-D) identify the blanks.

A. (1)

B. (2)

C. (3)

D. (4)

 

Q. 178 Given below is the representation of the extent of global diversity of invertebrates. What groups the four portions (A-D) represent respectively?

A. (1)

B. (2)

C. (3)

D. (4)

 

Q. 179 A scrubber in the exhaust of a chemical industrial plant removes:

A. Gases like sulphur dioxide

B. Particulate matter of the size 5 micrometer or above

C. Gases like ozone and methane

D. Particulate matter of the size 2.5 micrometer or less

 

Q. 180 If 20 J of energy is trapped at producer level, then how much energy will be available to peacock as food in the following chain?

Plant → mice → snake → peacock

A. 0.02 J

B. 0.002 J

C. 0.2 J

D. 0.0002 J

 

 

Answer Sheet 
Question 1 2 3 4 5 6 7 8 9 10
Answer D A D C C A B C D A
Question 11 12 13 14 15 16 17 18 19 20
Answer C A B C D A C D B A
Question 21 22 23 24 25 26 27 28 29 30
Answer D C C B D B B C C C
Question 31 32 33 34 35 36 37 38 39 40
Answer D D B B D C D B B B
Question 41 42 43 44 45 46 47 48 49 50
Answer C D C A D A D C D D
Question 51 52 53 54 55 56 57 58 59 60
Answer C C A C B D B D B A
Question 61 62 63 64 65 66 67 68 69 70
Answer A A C B C B A C C C
Question 71 72 73 74 75 76 77 78 79 80
Answer B B B B A D B D C D
Question 81 82 83 84 85 86 87 88 89 90
Answer D C D A C A B D A B
Question 91 92 93 94 95 96 97 98 99 100
Answer D A B A C D C B C C
Question  101 102 103 104 105 106 107 108 109 110
Answer B D D B B C A A A D
Question  111 112 113 114 115 116 117 118 119 120
Answer A C A A C B C D D B
Question 121 122 123 124 125 126 127 128 129 130
Answer A C B D A D D B C B
Question 131 132 133 134 135 136 137 138 139 140
Answer D D C C B D B B B C
Question 141 142 143 144 145 146 147 148 149 150
Answer C A B B B A C B A C
Question 151 152 153 154 155 156 157 158 159 160
Answer A A D A D C B C A B
Question 161 162 163 164 165 166 167 168 169 170
Answer C A B B C C A A B C
Question 171 172 173 174 175 176 177 178 179 180
Answer A A C C B D C D D A

SSC CGL Tier-I 26 June 2011 Shift-I Previous Year Paper

SSC CGL Tier 1 2011 26 Jun Shift 1

 

Q. 1  6:64::11:?

A. 144

B. 169

C. 121

D. 124

 

Q. 2  123:4::726:?

A. 23

B. 26

C. 14

D. 12

 

Q. 3 Fish:Scales::Bear:?

A. Feathers

B. Leaves

C. Fur

D. Skin

 

Q. 4 Writer:Pen::?

A. Needle:Tailor

B. Artist:Brush

C. Painter:Canvas

D. Teacher:Class

 

Q. 5 NUMERAL:UEALRMN::ALGEBRA:?

A. LRBAGEA

B. BARLAGE

C. LERAGBA

D. LERABGA

 

Q. 6 BDAC:FHEG::NPMO:?

A. RQTS

B. QTRC

C. TRQS

D. RTQS

 

Q. 7 FGHI:OPQR::BCDE:?

A. KLMJ

B. KLMN

C. IUVW

D. STUW

 

Q. 8 PNLJ:IGEC::VTRP:?

A. OMKI

B. RSTU

C. QSRC

D. QROM

 

Q. 9 19:60::20:?

A. 57

B. 69

C. 81

D. 93

 

Q. 10 Find the odd number from the given alternatives

A. 162

B. 405

C. 567

D. 644

 

Q. 11 Find the odd number from the given alternatives

A. 156

B. 201

C. 273

D. 645

 

Q. 12 Find the odd word from the given alternatives

A. Flute

B. Violin

C. Guitar

D. Sitar

 

Q. 13 Find the odd word from the given alternatives

A. Prod

B. Sap

C. Jab

D. Thrust

 

Q. 14 Find the odd word from the given alternatives

A. JKOP

B. MNST

C. CABD

D. OPWX

 

Q. 15 Find the odd word from the given alternatives

A. CAFD

B. TSWV

C. IGLJ

D. OMRP

 

Q. 16 In the following question, number of letters are skipped in between by particular rule. What of the following series observes the rules?

A. ABFGJK

B. ACFJOU

C. MPQSTV

D. ADFHJL

 

Q. 17 Find out the pair of numbers that does not belong to the group for lack of common property.

A. 11-115

B. 10-90

C. 9-72

D. 8-56

 

Q. 18 Which one of the following responses would be a meaning full descending order of the following?

1 Major

2 Captain

3 Colonel

4 Brigadier

5 Lt.General

A. 5,4,3,1,2

B. 5,1,4,2,3

C. 4,5,1,3,2

D. 3,4,2,5,1

 

Q. 19 Arrange the following words as per order in the dictionary

1 Dissident

2 Dissolve

3 Dissent

4 Dissolute

5 Dissolution

A. 3,1,4,5,2

B. 3,2,1,4,5

C. 3,1,4,2,5

D. 3,2,4,5,1

 

Q. 20 Which one set of letters when sequentially placed at the gaps in the given letter series shall complete it?

__cb__cab__baca__cba__ab

A. cabcb

B. abccb

C. bacbc

D. bcaba

 

Q. 21 Choose the correct alternatives from the given ones that will complete the series.

4,196,16,169,?,144,64

A. 21

B. 81

C. 36

D. 32

 

Q. 22 Choose the correct alternatives from the given ones that will complete the series.

8,15,36,99,288,……?….

A. 368

B. 676

C. 855

D. 908

 

Q. 23 Choose the correct alternatives from the given ones that will complete the series.

XYZCBAUVWFE….?…..?….

A. DR

B. RS

C. DS

D. MN

 

Q. 24 Choose the correct alternatives from the given ones that will complete the series.

reoc,pgme,nikg,lkii….?….

A. acef

B. jmgk

C. efgh

D. wxyz

 

Q. 25 Among her children, Ganga’s favourites are Ram and Rekha. Rekha is the mother of sharat, who is loved by his uncle mithun. The head of the family is Ram lal, who is succeeded by his sons Gopal and Mohan, Gopal and Ganga have been married for 35 years and have 3 children. What is the relation between Mithun and Mohan?

A. Uncle

B. Son

C. Brother

D. No relation

 

Q. 26 Find the wrong number in the series.

6,9,15,22,51,99

A. 99

B. 51

C. 22

D. 15

 

Q. 27 In a row of girls, Kamla is 9th from the left and Veena is 16th from the right. If they interchange their positions, Kamla becomes 25th from the left. How many girls are there in the row?

A. 34

B. 36

C. 40

D. 41

 

Q. 28 Ravi has spend a quarter of his life as a boy, one fifth as a youth, one third as a man and 13 years in old age, What is his present age?

A. 70 years

B. 80 years

C. 60 years

D. 65 years

 

Q. 29 Five boys A,B,C,D,E are sitting in a park in a circle. A is facing south-west, D is facing southeast, B and E are right opposite A and D respectively and C is equidistant between D and B. Which direction is C facing?

A. West

B. South

C. North

D. East

 

Q. 30 At what time are the hands of clocks together between 6 and 7?

A. 32 8/11 minutes past 6

B. 34 8/11 minutes past 6

C. 30 8/11 minutes past 6

D. 32 5/7 minutes past 6

 

Q. 31 Out of 100 families in the neighbourhood, 50 have radios, 75 have Tvs and 25 has VCRs. Only 10 families has all 3 and each VCR owner also has a TV. If some families have radio only, how many have TV only?

A. 30

B. 35

C. 40

D. 45

 

Q. 32 Suresh was born on 4th october 1999. Shashikanth was born 6 days before Suresh. The independence day of that year fell on Sunday. Which day was Shashikanth born?

A. Tuesday

B. Wednesday

C. Monday

D. Sunday

 

Q. 33 From the given alternative words, select the word which cannot be formed using the letters of the given word:

‘CONCENTRATION’

A. CONCERN

B. NATION

C. TRAIN

D. CENTRE

 

Q. 34 In a certain office, 1/3 of the workers are women, 1/2 of the mend are married and 1/3 of the married women have children. If 3/4 men are married and 2/3 men have children, then what part of workers is without children?

A. 5/18

B. 4/9

C. 11/18

D. 17/36

 

Q. 35 If a man on a moped starts from a point and 4 km south, then turns and rides 2 km to turn again to the right to ride 4 km more, towards which direction is he moving?

A. North

B. West

C. East

D. South

 

Q. 36 Ganesh cycles towards south-west a distance of 8 m, then he moves towards east a distance of 20 m, from there he moves towards north-east a distance of 8 m, then he moves towards west a distance of 6 m. From there he moves towards north-east a distance of 2 m. Then he moves towards west a distance of 4 m and then he moves towards southwest a distance of 2 m and stops at that point. How far is he from the starting point?

A. 12 m

B. 10 m

C. 8 m

D. 6 m

 

Q. 37 Two statements are given by 4 inferences. Select the alternative which is most appropriate.

Statements:

1 India is becoming industrial.

2 Pollution is a problem associated with industrialization.

Inferences:

1 All industrial centers are polluted.

2 India is polluted

3 Polluted nations are industrialized

4 India may become polluted

A. All are appropriate

B. None is appropriate

C. only 4 is appropriate

D. Only 2 is appropriate

 

Q. 38 From the given alternative words, select the word which can be formed using the letters of the given word:

‘DETERMINATION’

A. DECLARATION

B. NATIONAL

C. TERMINATED

D. DEVIATED

 

Q. 39 If in a certain code HYDROGEN is written as JCJZYSSD, then how ANTIMONY be written in that code?

A. CPVKOQPA

B. CRZQWABO

C. ERXMQSRC

D. GTZOSUTE

 

Q. 40 If DELHI is coded as 73541 and CALCUTTA as 82589662 then how can CALICUT coded?

A. 5279431

B. 5978013

C. 8251896

D. 8543691

 

Q. 41 Select the missing numbers from the given responses

A. 8

B. 3

C. 6

D. 36

 

Q. 42 Select the missing numbers from the given responses

A. 60

B. 68

C. 55

D. 65

 

Q. 43 How many rectangles are there in the given diagram?

A. 4

B. 7

C. 9

D. 18

 

Q. 44 Which of the following diagrams represents the relationship among Sun , Moon and Star?

A. a

B. b

C. c

D. d

 

Q. 45 Some equations have been solved on the basis of a certain system. Find the correct answer for the unsolved equation on that basis.

If 9*7 = 32, 13*7 = 120, 17*9 = 208, then 19*11 = ?

A. 150

B. 180

C. 210

D. 240

 

Q. 46 Forecast the growth rate for the year 1995 from the following data:

A. 7.8

B. 8.6

C. 9.7

D. 9.9

 

Q. 47 If ‘-‘ stands for ‘/’ , ‘+’ stand for ‘*’, ‘/’ for ‘-‘ and ‘*’ for ‘+’, which one of the following equation is correct?

A. 30-6+5*4/2 = 27

B. 30+6-5/4*2 = 30

C. 30*6/5-4+2 = 32

D. 30/6*5+4-2 = 40

 

Q. 48 From the given answer figures, select the one in which the questions figure is hidden/embedded.

A. a

B. b

C. c

D. d

 

Q. 49 If a mirror is placed on the line MN, then which of the answer figures is the correct image of the given question figures?

A. a

B. b

C. c

D. d

 

Q. 50 A piece of paper is folded and cut as shown below in the question figures. From the given answer figures, indicate how it will appear when opened.

A. a

B. b

C. c

D. d

 

Q. 51 Judicial review in the indian constitution is based on :

A. Rule of law

B. Due process of law

C. Procedure established by law

D. Precedents and conventions

 

Q. 52 The drafting of the constitution was completed on:

A. 26th jan 1950

B. 26th dec 1949

C. 26th nov 1949

D. 30th nov 1949

 

Q. 53 Who was the president of the constitution assembly?

A. pt. Jawahar Lal Nehru

B. Sardar patel

C. Dr. Rajendra Prasad

D. Dr. B.R.Ambedkar

 

Q. 54 Which innovative discussion process is introduced by the Indian parliament to the world parliamentary system?

A. Question hour

B. Zero hour

C. Resolutions

D. Presidential speech

 

Q. 55 The judges of the supreme court retire at the age of:

A. 60 years

B. 65 years

C. 62 years

D. 58 years

 

Q. 56 When there is an official change in the exchange rate of domestic currency, then it is called:

A. Appreciation

B. Depreciation

C. Revalution

D. Deflation

 

Q. 57 Inflation redistributes income and wealth in favour of:

A. Pensioners

B. Poor

C. Middle class

D. rich

 

Q. 58 The fringe benefit tax was introduced in the budget of:

A. 2003-04

B. 2004-05

C. 2005-06

D. 2006-07

 

Q. 59 In the budget estimates for 2011-12, an allocation of: 400 crore has been made to bring in green revolution in the east in the cropping system of:

A. Wheat

B. Rice

C. Jowar

D. Pulses

 

Q. 60 As announced by finance minister in his budget speech on 28-2-2011, the income tax exemption limit for individuals other than senior citizens and women has been raised to

A. Rs. 1,80,000

B. Rs. 1,90,000

C. Rs. 2,00,000

D. Rs. 2,50,000

 

Q. 61 Who among the following british persons, admitted the revolt of 1857 as a national revolt?

A. Lord Dalhousie

B. Lord Canning

C. Lord Ellenborough

D. Lord Disraeli

 

Q. 62 The greek ambassador sent to chandragupta maurya’s court was:

A. Kautilya

B. Seleucus Nicator

C. Megasthenes

D. Justin

 

Q. 63 Identify the European power from whom shivaji obtained cannons and ammunitions

A. The french

B. The Portuguese

C. The dutch

D. The English

 

Q. 64 The call of “Back to vedas” was given by

A. Swami vivekananda

B. swami dayanand saraswati

C. aurobindo ghosh

D. raja ram mohan roy

 

Q. 65 Simon commission was boycotted by the, nationalist leaders of india because

A. they felt it was only an eyewash

B. all the members of the commission were english

C. the members of the commissions were biased against india

D. it did not meet the demands of indians

 

Q. 66 The lowest layer of atmosphere is

A. Stratosphere

B. thermosphere

C. Troposphere

D. Mesosphere

 

Q. 67 The konkan railway connects

A. Goa – mangalore

B. Roha – mangalore

C. Kanyakumari – mangalore

D. Kanyakumari – Mumbai

 

Q. 68 Bark of this tree is used as a condiment

A. Cinnamon

B. Clove

C. Neem

D. Palm

 

Q. 69 How much of the earth’s land surface is desert?

A. 1/10th

B. 1/5th

C. 1/3rd

D. 1/6th

 

Q. 70 Which of the following is called the ‘shrimp capital of India’?

A. Mangalore

B. Nagapatnam

C. Kochi

D. Nellore

 

Q. 71 River indus originates from

A. Hindukush range

B. Himalayan range

C. Karakoram range

D. Kailash range

 

Q. 72 The atmospheric air is held to the earth by

A. gravity

B. winds

C. cloud

D. rotation of earth

 

Q. 73 The function of ball bearings in a ring is

A. to increase friction

B. to convert kinetic friction into rolling friction

C. to convert static friction into kinetic friction

D. just for convenience

 

Q. 74 ‘Shock – absorbers’ are usually made of steel as it

A. is not brittle

B. has lower elasticity

C. has higher elasticity

D. hasno ductile property

 

Q. 75 The first computer made available for commercial use was

A. MANIAC

B. ENIAC

C. UNIVAC

D. EDSAC

 

Q. 76 A communications network which is used by large organisations over regional, national or global area is called

A. LAN

B. WAN

C. MAN

D. VAN

 

Q. 77 Who was the architecture of north and south blocks of the central secretariat in Delhi?

A. Sir edward lutyens

B. Herbert Baker

C. Robert tor Russell

D. Antonin Raymond

 

Q. 78 Saliva helps in the digestion of

A. Fats

B. Starch

C. Proteins

D. Vitamins

 

Q. 79 The longest bone in the human body is

A. Ulna

B. Humerus

C. Femur

D. Tibia

 

Q. 80 Red data book gives information about species which are

A. extinct

B. endangered

C. dangerous

D. rare

 

Q. 81 Which of the following is the smallest bird?

A. pigeon

B. parrot

C. Humming bird

D. Houise sparrow

 

Q. 82 The time period of a pendulum when taken to the moon would

A. remain the same

B. decrease

C. become zero

D. increase

 

Q. 83 Indian army’s operation ‘Saiyam’ was related to

A. Kashmir

B. Indo-China border in central region

C. North-East

D. Indo-pak border in punjab and rajasthan

 

Q. 84 Which of the following can be used as fuel in propellant or rockets?

A. Liquid Hydrogen + Liquid Nitrogen

B. Liquid oxygen + Liquid argon

C. Liquid nitrogen + Liquid oxygen

D. Liquid hydrogen + Liquid oxygen

 

Q. 85 The addition of gypsum of portland cement helps in

A. increasing the strength of cement

B. rapid setting of cement

C. preventing rapid setting of cement

D. reduction in the cost of cement

 

Q. 86 White lung disease is prevalent among the workers of

A. paper industry

B. cement industry

C. Cotton industry

D. pesticide industry

 

Q. 87 Iodoform is used as an

A. antipyretic

B. analgestic

C. antiseptic

D. anaesthetic

 

Q. 88 An artificial ecosystem is represented by

A. pisciculture tank

B. agricultural land

C. zoo

D. aquarium

 

Q. 89 The constituents of automobile exhaust that can cause cancer are

A. Oxides of nitrogen

B. Carbon monoxide

C. Polycyclic hydrocarbon

D. lead

 

Q. 90 The optimum dissolved oxygen level required for survival of aquatic organisms is

A. 4 – 6

B. 2 – 4

C. 8 – 10

D. 12 – 16

 

Q. 91 The world’s only floating national park is situated in

A. Manipur

B. Kuala Lumpur

C. Bilaspur

D. Dispur

 

Q. 92 According to the latest ‘Education development Index’ based on a series of surveys conducted by the national university of education, planning and administration and released in feb 2011, the state with the highest development index is

A. Tamil naidu

B. Kerala

C. Punjab

D. Gujarat

 

Q. 93 The ‘project snow leopard’ to conserve the endangered species, launched by the union ministry of environment and forests covers the states of

A. Jammu & Kashmir and Himachal pradesh only

B. Jammu & Kashmir and Himachal pradesh and uttarakhand only

C. Jammu & Kashmir, Himachal pradesh, uttarakhand and arunachal pradesh only

D. Jammu & Kashmir, Himachal pradesh, uttarakhand and arunachal pradesh and sikkim

 

Q. 94 In the railway budget for 2011-2012, an outlay of Rs.57,630 crore has been announced for adding new lines with length of

A. 1000 km

B. 1200 km

C. 1300 km

D. 1500 km

 

Q. 95 The strong earthquake-cum-tsunami which has moved japan’s main island by a few feet and has caused the earth’s axis to wobble is expected to lead to the shortening of the day by

A. 1.6 microseconds

B. 2.3 microseconds

C. 3.1 microseconds

D. 3.4 microseconds

 

Q. 96 Which of the following folk/tribal dances is associated with uttar pradesh

A. veedhi

B. thora

C. tamasha

D. rauf

 

Q. 97 Which of the following books has been written by atiq rahimi

A. earth and ashes

B. This savage rite

C. The red devil

D. Witness the night

 

Q. 98 Who is the recipient of the sahitya akademi award 2010 in hindi literatur category?

A. uday prakash

B. laxman dubey

C. nanjil nandan

D. mangat badal

 

Q. 99 The 2010 FIFA World cup final was held at

A. Paris

B. Berlin

C. Johannesburg

D. London

 

Q. 100 Who received sangeet natak akademi’s ustad bismillah khan yuva puruskar for 2009 in ‘hindustani vocal music’?

A. Omkar shrikant dadarkar

B. ragini chander shekar

C. abanti chakravorty and sukracharya rabha

D. k. nellai Manikandan

 

Q. 101 The value of given equation is

A. 4

B. 0

C. √2

D. 3√6

 

Q. 102 What is the answer of the given equation

A. 2.3

B. 3

C. 6

D. 6.3

 

Q. 103 What is the square root of the given equation

A. √3 + √2

B. √3 – √2

C. √2 +_ √3

D. √2 – √3

 

Q. 104 The remainder when 3²¹ is divided is

A. 1

B. 2

C. 3

D. 4

 

Q. 105 What is the value of the given equation?

A. 38/109

B. 109/38

C. 1

D. 116/109

 

Q. 106 The last digit of (1001)²⁰⁰⁸ + 1002 is

A. 0

B. 3

C. 4

D. 6

 

Q. 107 The given equation is equal to?

A. 5/9

B. 1-1/7

C. 4/7

D. 1-2/7

 

Q. 108 If the sum of two numbers be multiplied by each number separately, the product so obtained are 247 and 114. The sum of the numbers is

A. 19

B. 20

C. 21

D. 23

 

Q. 109 Find a number, one-seventh of which exceeds its eleventh part by 100

A. 1925

B. 1825

C. 1540

D. 1340

 

Q. 110 If x*y = (x+3)²(y-1) then the value of 5 * 4 is

A. 192

B. 182

C. 180

D. 172

 

Q. 111 Answer of the given equation is

A. 10

B. 100

C. 1000

D. none of these

 

Q. 112 If 9√x = √12 + √147 then x = ?

A. 2

B. 3

C. 4

D. 5

 

Q. 113 A and B can complete a piece of work in 8 days, B and C can do it in 12 days, C and A can do it in 8 days.A,B and C together can complete it in

A. 4 days

B. 5 days

C. 6 days

D. 7 days

 

Q. 114 X is 3 times as fast as Y and is able to complete the work in 40 days less then Y. Then the time in which they can complete the work together is

A. 15 days

B. 10 days

C. 7.5 days

D. 5 days

 

Q. 115 A copper wire is bent in the shape of a square of area 81 cm². If the same wire is bent in the form of a semicircle, the radius of the semicircle is (take pie = 22/7)

A. 16

B. 14

C. 10

D. 7

 

Q. 116 The volume (in m³) of rain water that can be collected from 1.5 hectares of ground in a rainfall of 5 cm is

A. 75

B. 750

C. 7500

D. 75000

 

Q. 117 A river 3 m deep and 40 m wide is flowing at the rate of 2 km per hour. How much water (in liters) will fall in the sea in a minute?

A. 4,00,000

B. 40,00,000

C. 40,000

D. 4,000

 

Q. 118 The L.C.M of three different number is 120. Which of the following cannot be their H.C.F?

A. 8

B. 12

C. 24

D. 35

 

Q. 119 A number when divided by 49 leaves 32 as reminder. This number when divided by 7 will have the reminder as

A. 2

B. 3

C. 2

D. 5

 

Q. 120 In an examination a student scores 4 marks for every correct answers and loses 1 mark for every wrong answer. If he attempts all 75 questions and secures 125 marks, the number of questions he attempted correctly is

A. 35

B. 40

C. 42

D. 46

 

Q. 121 The traffic lights at three different road crossings changes after 24 seconds, 36 seconds and 54 seconds respectively. If they all change simultaneously at 10:15:00 am, then at what time will then change again

A. 10:16:54 am

B. 10:18:36 am

C. 10:17:02 am

D. 10:22:12 am

 

Q. 122 A can do a work in 12 days. When he had worked for 3 days, B joined him. If they can complete the work in 3 days, in how many days can B alone finish the work?

A. 6 days

B. 12 days

C. 4 days

D. 8 days

 

Q. 123 Among three numbers, the first is twice the second and thrice the third. If the average of the three numbers is 49.5, then the difference between the first and the third number is

A. 54

B. 28

C. 39.5

D. 41.5

 

Q. 124 The mean of 50 numbers is 30. Later it was discovered that two entries were wrongly entered as 82 and 13 instead of 28 and 31. Find the correct mean.

A. 36.12

B. 30.66

C. 29.28

D. 38.21

 

Q. 125 A bicycle wheel makes 5000 revolutions in moving 11 km. Then the radius of the wheel (in am) is (take pie = 22/7)

A. 70

B. 35

C. 17.5

D. 140

 

Q. 126 The perimeter of a triangle is 40 cm and its area is 60 cm². If the largest side measures 17cm, then the length (in cm ) of the smallest side of the triangle is

A. 4

B. 6

C. 8

D. 15

 

Q. 127 A shopkeeper allows a discount of 10 % to his customers and still gains 20%. Find the market price of the article which costs?

A. Rs 600

B. Rs 540

C. Rs 660

D. 580

 

Q. 128 What single discount is equivalent to two successive discounts of 20 % and 15%?

A. 35%

B. 32%

C. 34%

D. 30%

 

Q. 129 In a business partnership among A,B,C and D the profit is hard as follows:

What is the total profit

A. Rs 21,12,500

B. Rs 1,37,500

C. Rs 90,000

D. Rs 2,70,000

 

Q. 130 A can contains a mixture of two liquids A and B in ratio 7:5. When 9 liters of mixture are drawn off the can is filled with B, the ratio of A and B becomes 7:9. Liters of liquid A contained by the can initially was

A. 10

B. 20

C. 21

D. 25

 

Q. 131 What number should be added to or subtracted from each term of the ratio 17 : 24 so that it becomes equal to 1 : 2?

A. 5 is subtracted

B. 10 is added

C. 7 is added

D. 10 is subtracted

 

Q. 132 The ratio of weekly income of A and B is 9 : 7 and the ratio of their expenditures is 4 : 3. If each saves Rs. 200 per week, then the sum of their weekly income is?

A. Rs 3600

B. Rs 4200

C. Rs 4800

D. Rs 5600

 

Q. 133 If 30% of A is added to 40% of B, the answer is 80 % of B. What percent of A is B?

A. 30%

B. 40%

C. 70%

D. 75%

 

Q. 134 A man can row 6 km/h in still water. If the speed of current is 2 km/h , it takes 3 hours more in upstream than in downstream for the same distance. The distance is

A. 30 km

B. 24 km

C. 20 km

D. 32 km

 

Q. 135 If the selling price of 10 articles is equal to the cost price of 11 articles, then the gain percentage is

A. 10

B. 11

C. 15

D. 25

 

Q. 136 While selling a watch, a shopkeeper gives discount of 5%. If he gives discount of 6%, he earns Rs.15 less as profit. What is the marked price of the watch?

A. Rs 1250

B. Rs 1400

C. Rs 1500

D. Rs 750

 

Q. 137 Krishna purchased a number of articles for Rs.10 for each and the same number for Rs.14 each. He mixed them together and sold them for Rs.13 each. Then his gain or loss percent is :

A. Loss 8 1/3 %

B. Gain 8 2/3%

C. Loss 8 2/3%

D. Gain 8 1/3%

 

Q. 138 A trader bought two horses for Rs.19500. He sold one at a loss of 20% and other at a profit of 15%. If the selling price of each horse is the same then the cost price of them respectively are

A. Rs.10000 and Rs.9500

B. Rs.11500 and Rs.8000

C. Rs.12000 and Rs.7500

D. Rs.10500 and Rs.9000

 

Q. 139 The cost price of an article is 40% of the selling price. What percent of the cost price is the selling price?

A. 140%

B. 200%

C. 220%

D. 250%

 

Q. 140 IF 90% of A = 30% of B and B = 2x% of A, then the value of x is

A. 450

B. 400

C. 300

D. 150

 

Q. 141 When the price of sugar decreases by 10%, a man could by 1 kg more for Rs.270. Then the original price of sugar per kg is

A. Rs. 25

B. Rs. 30

C. Rs. 27

D. Rs. 32

 

Q. 142 If the price of sugar is raised by 25%, find by how much percent a householder must reduce his consumption of sugar so as not to increase his expenditure?

A. 10

B. 20

C. 18

D. 25

 

Q. 143 The difference between the compound interest and simple interest on Rs. 10000 for two years is Rs. 25. The rate of interest per annum is

A. 5%

B. 7%

C. 10%

D. 12%

 

Q. 144 A student goes to school at the rate of 2-1/2 km/h and reaches 6 minutes late. If he travels at the speed of 3km/h, he is 10 minutes early. The distance between the school and his house is (in km)

A. 5

B. 4

C. 3

D. 1

 

Q. 145 A sum of money placed at compound interest doubles itself in 4 years. In how many years will it amount to 4 times itself?

A. 12 years

B. 13 years

C. 8 years

D. 16 years

 

Q. 146 The simple interest on a sum for 5 years is one-fourth of the sum. The rate of interest per annum is

A. 5%

B. 6%

C. 4%

D. 8%

 

Q. 147 The following graph shows the demand and production of cotton by 5 companies. Observe it and answer the question

The production of company D is how many times that of the production of company A?

A. 1.8

B. 1.5

C. 0.5

D. 0.4

 

Q. 148 The following graph shows the demand and production of cotton by 5 companies. Observe it and answer the question

The demand of company B is what percent of demand for company C?

A. 1.5

B. 2.5

C. 25

D. 30

 

Q. 149 The following graph shows the demand and production of cotton by 5 companies. Observe it and answer the question

What is the ratio of companies having more demand than production to those having more production then demand?

A. 2:3

B. 4:1

C. 3:2

D. 1:4

 

Q. 150 The following graph shows the demand and production of cotton by 5 companies. Observe it and answer the question

What is the difference (in tonnes) between average demand and average production of the five companies taken together?

A. 320

B. 420

C. 2100

D. 1050

 

Q. 151 Mark the option which has error, if not then mark the option d

A. His son

B. is working

C. very hardly

D. no error

 

Q. 152 Mark the option which has error, if not then mark the option d

A. Do you know that it was I

B. who has done

C. this piece of beautiful work?

D. no error

 

Q. 153 Mark the option which has error, if not then mark the option d

A. The company has ordered

B. some

C. new equipments

D. no error

 

Q. 154 Mark the option which has error, if not then mark the option d

A. The future of food companies

B. seems quite secure

C. owed to ever growing demand

D. no error

 

Q. 155 Mark the option which has error, if not then mark the option d

A. The vaccine

B. When hit the indian market

C. is dogged by controversy

D. no error

 

Q. 156 If you had followed the rules, you _______ disqualified.

A. will not be

B. would not be

C. will not have been

D. would not have been

 

Q. 157 The housewife ____ the cakes burning, and ran to switch off the oven

A. Smell

B. Smells

C. Smelt

D. Smelling

 

Q. 158 _____ an old legend, King Shirharm lived in India.

A. In the event of

B. Due to

C. According to

D. In reference to

 

Q. 159 _____ you leave no, you will be late.

A. Until

B. Till

C. Unless

D. Although

 

Q. 160 The ____ were arrested for illegally hunting the bears.

A. Poachers

B. rangers

C. soldiers

D. villagers

 

Q. 161 Choose the word which give the correct meaning of the given word

GENIAL

A. cordial

B. Unselfish

C. Careful

D. Specific

 

Q. 162 Choose the word which give the correct meaning of the given word

ACCRUE

A. Accumluate

B. Accommodate

C. Grow

D. Suffice

 

Q. 163 Choose the word which give the correct meaning of the given word

LOQUACIOUS

A. Talkative

B. slow

C. Content

D. Unclear

 

Q. 164 Choose the word which give the correct meaning of the given word

VINDICTIVE

A. Imaginative

B. Accusative

C. Spiteful

D. Aggressive

 

Q. 165 Choose the word which give the correct meaning of the given word

INCLEMENT

A. Selfish

B. Active

C. Unfavorable

D. Inactive

 

Q. 166 Choose the opposite of the given word from the given options

SYNTHETIC

A. Natural

B. Plastic

C. Cosmetic

D. Apathetic

 

Q. 167 Choose the opposite of the given word from the given options

ACCORD

A. Disagreement

B. Welcome

C. Disrespect

D. Confromity

 

Q. 168 Choose the opposite of the given word from the given options

INFIRMITY

A. Employment

B. Indisposition

C. Strength

D. Weakness

 

Q. 169 Choose the opposite of the given word from the given options

FEASIBLE

A. Useful

B. Impractical

C. Uneven

D. Important

 

Q. 170 Choose the opposite of the given word from the given options

METICULOUS

A. Forgetful

B. Destructive

C. Careless

D. Flagrant

 

Q. 171 Choose the alternative which gives the meaning for the given phrase

To be all at sea

A. a family voyage

B. lost and confused

C. in the middle of the ocean

D. a string of islands

 

Q. 172 Choose the alternative which gives the meaning for the given phrase

TO take to one’s heels

A. to walk slowly

B. to run away

C. to march forward

D. to hop and jump

 

Q. 173 Choose the alternative which gives the meaning for the given phrase

To bite the dust

A. cat voraciously

B. have nothing to eat

C. cat roots

D. none of the above

 

Q. 174 Choose the alternative which gives the meaning for the given phrase

A bolt from the blue

A. a delayed event

B. an inexplicable event

C. an unexpected event

D. an unpleasant event

 

Q. 175 Choose the alternative which gives the meaning for the given phrase

Cold comfort

A. absurdity

B. deception

C. slight satisfaction

D. foolish proposal

 

Q. 176 Select the proper replacement of the part written in capital.

My friend lives in a nearby street WHOSE NAME i have forgotten

A. the name of which

B. which name

C. of which name

D. no improvement

 

Q. 177 Select the proper replacement of the part written in capital.

HE BOTH WON A MEDAL AND A SCHOLARSHIP

A. He won a medal and a scholarship both

B. Both he won a medal and a scholarship

C. He won both a medal and a scholarship

D. No improvement

 

Q. 178 Select the proper replacement of the part written in capital.

HE HAS FOR GOOD LEFT INDIA

A. He has left for good

B. He has left india for good

C. Good he has left india

D. No improvement

 

Q. 179 Select the proper replacement of the part written in capital.

WE ARE CREDIBLY INFORMED THAT THE MURDERER HAS GIVEN HIMSELF UP.

A. We are informed that the murderer has credibly given himself up

B. We are informed that the murderer has given credibly himself up

C. We are informed that credibly the murderer has given up himself

D. No improvement

 

Q. 180 Select the proper replacement of the part written in capital.

We generally select ONE OF THE MOST INTELLIGENT STUDENT OF THE SCHOOL for this award

A. one of the most intelligent student of school

B. one of the intelligent most student of school

C. one of the intelligent most students of the school

D. no improvement

 

Q. 181 Choose the word which can be substituted for given sentence

Pertaining to cattle

A. Canine

B. Feline

C. Bovine

D. Verminous

 

Q. 182 Choose the word which can be substituted for given sentence

To look at someone in an angry or threatening way

A. Glower

B. Gnaw

C. Gnash

D. Grind

 

Q. 183 Choose the word which can be substituted for given sentence

A post with little work but high salary

A. Director

B. Trustee

C. Sinecure

D. Ombudsman

 

Q. 184 Choose the word which can be substituted for given sentence

Something that causes death

A. Dangerous

B. Fatal

C. Brutal

D. Horrible

 

Q. 185 Choose the word which can be substituted for given sentence

A person who writes decoratively

A. Calligrapher

B. Collier

C. Choreographer

D. Cartographer

 

Q. 186 From the given words in the options, choose the one with correct spelling

A. malaign

B. arraign

C. asigne

D. degine

 

Q. 187 From the given words in the options, choose the one with correct spelling

A. harrassment

B. embarrasment

C. fulfillment

D. denoument

 

Q. 188 From the given words in the options, choose the one with correct spelling

A. agnostik

B. accomplice

C. advercity

D. acrimonous

 

Q. 189 From the given words in the options, choose the one with correct spelling

A. dysentery

B. momentary

C. cementary

D. comentary

 

Q. 190 From the given words in the options, choose the one with correct spelling

A. ebulient

B. jubilant

C. iminent

D. tolemt

 

Q. 191 Some words have been left in the passage, fill in the blanks with the help of alternatives given

A. problem

B. question

C. matter

D. query

 

Q. 192 Some words have been left in the passage, fill in the blanks with the help of alternatives given

A. around

B. out

C. about

D. on

 

Q. 193 Some words have been left in the passage, fill in the blanks with the help of alternatives given

A. since

B. during

C. around

D. from

 

Q. 194 Some words have been left in the passage, fill in the blanks with the help of alternatives given

A. ideas

B. opinions

C. stories

D. matters

 

Q. 195 Some words have been left in the passage, fill in the blanks with the help of alternatives given

A. solution

B. novel

C. book

D. answer

 

Q. 196 Some words have been left in the passage, fill in the blanks with the help of alternatives given

A. read

B. think

C. open

D. guess

 

Q. 197 Some words have been left in the passage, fill in the blanks with the help of alternatives given

A. now

B. time

C. then

D. ago

 

Q. 198 Some words have been left in the passage, fill in the blanks with the help of alternatives given

A. moon

B. time

C. earth

D. mars

 

Q. 199 Some words have been left in the passage, fill in the blanks with the help of alternatives given

A. part

B. division

C. opening

D. center

 

Q. 200 Some words have been left in the passage, fill in the blanks with the help of alternatives given

A. end

B. begin

C. think

D. work

 

 

Answer Sheet
Question 1 2 3 4 5 6 7 8 9 10
Answer B D C C C D B A B D
Question 11 12 13 14 15 16 17 18 19 20
Answer A A B C B B A A A C
Question 21 22 23 24 25 26 27 28 29 30
Answer C C A B D C C C D A
Question 31 32 33 34 35 36 37 38 39 40
Answer C B D C D B C C A C
Question 41 42 43 44 45 46 47 48 49 50
Answer C D D D D C A C C C
Question 51 52 53 54 55 56 57 58 59 60
Answer C C C C B C D C B A
Question 61 62 63 64 65 66 67 68 69 70
Answer D C B B B C B A B D
Question 71 72 73 74 75 76 77 78 79 80
Answer D A B C C B B B C B
Question 81 82 83 84 85 86 87 88 89 90
Answer C D D D B D C D D A
Question 91 92 93 94 95 96 97 98 99 100
Answer A B D C C B A A C A
Question 101 102 103 104 105 106 107 108 109 110
Answer B B A C A B B A A A
Question 111 112 113 114 115 116 117 118 119 120
Answer B B C A D B D D A
Question 121 122 123 124 125 126 127 128 129 130
Answer B A A C B C A B C C
Question 131 132 133 134 135 136 137 138 139 140
Answer D C D B A C D B D D
Question 141 142 143 144 145 146 147 148 149 150
Answer B B A B C A A C C B
Question 151 152 153 154 155 156 157 158 159 160
Answer C B D C C D C C C A
Question 161 162 163 164 165 166 167 168 169 170
Answer A A A C C A A C B C
Question 171 172 173 174 175 176 177 178 179 180
Answer B B D C C D C B A A
Question 181 182 183 184 185 186 187 188 189 190
Answer C A C B A B C B A B
Question 191 192 193 194 195 196 197 198 199 200
Answer C C D B D B D C A C

SSC CGL Tier-I 16 May 2010 Shift-I Previous Year Paper

SSC CGL Tier 1 2010 16 May Shift 1

Directions (1-8): In each of the following questions, select the related letters/word/number from the given alternatives.

Q. 1 ACEG : SUWY :: BDFH : ?

A. TVZX

B. RTZV

C. TVXZ

D. RTVZ

 

Q. 2 M/AC : N/AD :: O/AE : ?

A. P/AF

B. Q/AB

C. P/AC

D. R/AD

 

Q. 3 5 : 27 :: 9 : ?

A. 83

B. 81

C. 36

D. 18

 

Q. 4 6 : 11 :: 11 : ?

A. 6

B. 17

C. 20

D. 30

 

Q. 5 ABE : 8 :: KLO : ?

A. 37

B. 39

C. 38

D. 36

 

Q. 6 Sty : Pig : : Byre : ?

A. Eagle

B. Cow

C. Tiger

D. Hen

 

Q. 7 Patrol : Security : : Insurance : ?

A. Money

B. Policy

C. Savings

D. Risk

 

Q. 8 ADBC : EHFG :: ILJK : ?

A. MOPN

B. MPNO

C. ORPQ

D. MPON

 

Q. 9 Select the different one.

A. 25 , 36

B. 144 , 169

C. 100 , 121

D. 9 , 64

 

Q. 10 Select the different one.

A. heat

B. light

C. bulb

D. electricity

 

Q. 11 Select the different one.

A. wave

B. current

C. tide

D. storm

 

Q. 12 Select the different one.

A. X

B. Y

C. H

D. D

 

Q. 13 Select the different one.

A. ZKXJ

B. CMAL

C. TGRF

D. FRTK

 

Q. 14 Select the different one.

A. ABJNM

B. QRTUZ

C. IXYOQ

D. WFGOP

 

Q. 15 Arrange the following words according to the dictionary ?

1. Inventory

2. Involuntary

3. Invisible

4. Invariable

5. Investigate

A. 4 , 2 , 5 , 3 , 1

B. 4 , 5 , 1 , 3 , 2

C. 2 , 5 , 4 , 1 , 3

D. 4 , 1 , 5 , 3 , 2

 

Q. 16 Find the next two letters in the given series. B C E H L ?

A. XY

B. MN

C. QW

D. OP

 

Q. 17 Which one set of letters when sequentially placed at the gaps in the given letter series shall complete it ?

a_ b_ a_ _n_bb_abbn

A. abnabb

B. bnbban

C. bnbbna

D. babban

 

Q. 18 Find out a set of numbers amongst the four sets of numbers given in the alternatives, which is the most similar to the numbers given in the question. Given : (6, 30, 90)

A. 6 , 42 , 86

B. 7 , 42 , 218

C. 6 , 24 , 70

D. 8 , 48 , 192

 

Q. 19 Which one number is wrong in the given series?

126, 98, 70, 41, 14

A. 98

B. 70

C. 126

D. 41

 

Q. 20 Arrange the following in the meaningful/logical order

1. Exhaust

2. Night

3. Day

4. Sleep

5. Work

A. 1 , 3 , 5 , 2 , 4

B. 3 , 5 , 1 , 4 , 2

C. 3 , 5 , 1 , 2 , 4

D. 3 , 5 , 2 , 1 , 4

 

Q. 21   3, 4, 7, 11, 18, 29,…?…

A. 31

B. 39

C. 43

D. 47

 

Q. 22 AGMSY, CIOUA, EKQWC,…?… IOUAG, KQWCI

A. GMSYE

B. FMSYE

C. GNSYD

D. FMYES

 

Q. 23   975, 864, 753, 642,…?…

A. 431

B. 314

C. 531

D. 532

 

Q. 24   8, 24, 12,…?…18,54

A. 28

B. 36

C. 46

D. 38

 

Q. 25 Ashok’s mother was 3 times as old as Ashok 5 years ago. After 5 years she will be twice as old as Ashok. How old is Ashok today ?

A. 10 years

B. 15 years

C. 20 years

D. 25 years

 

Q. 26 M is the son of P. Q is the grand daughter of 0 who is the husband of P. How is M related to O?

A. son

B. daughter

C. mother

D. father

 

Q. 27 In a row of boys, Srinath is 7th from the left and Venkat is 12th from the right. If they interchange their positions, Srinath becomes 22nd from the left. How many boys are there in the row ?

A. 19

B. 31

C. 33

D. 34

 

Q. 28 From the given alternative words, select the word which cannot be formed using the letters of the given word Given :

IMPASSIONABLE

A. IMPASSABLE

B. IMPOSSIBLE

C. IMPASSIVE

D. IMPASSION

 

Q. 29 Only one meaningful word can be formed by rearranging the letter of the given jumbled word. Find out that word.

Given : MUSPOPAPOTIH

A. METAMORPHIC

B. PHILANTHROPIST

C. HIPPOCAMPUS

D. HIPPOPOTAMUS

 

Q. 30 Which number is wrong in the given series ?

1, 9, 25, 50, 81

A. 1

B. 25

C. 50

D. 81

 

Q. 31 A bus leaves Delhi with half the number of women as men. A Meerut, ten men get down and live women get in. Now there are equa number of men and women. Ho many passengers boarded the bus initially at Delhi ?

A. 36

B. 45

C. 15

D. 30

 

Q. 32 If the day before yesterday wa Sunday, what day will it be thre days after the day after tomorrow

A. Sunday

B. Monday

C. Wednesday

D. Saturday

 

Q. 33 Directions (33): In the following question, one statement is given followed by two assumptions I and II. You have to consider the statement to be true even if it seems to be at variance from commonly known facts. You have to decide which of the given

assumptions, if any, follow from the given statement.

Statement: Politicians become rich by the votes of the people.

Assumptions: I. People vote to make politician rich.

II. Politicians become rich by their virtue.

A. Only I is implicit

B. Only II is implicit

C. Both I and II are implicit

D. Both I and II are not implicit

 

Q. 34 Directions (34): In the following question, two statements P and Q are given followed by four conclusions I II, III and IV. You have to consider the two statements to be true even if the seem to be at variance from commonly known facts. You have to decide whit of the given conclusions, if any, folio the given statements.

Statements:

P. All men are women.

Q. All women are crazy.

Conclusions :

I. All men are crazy.

II. All the crazy are men.

III. Some of the crazy are men.

IV. Some of the crazy are women

A. None of the conclusions follow

B. All the conclusions follow

C. Only I, III and IV follow

D. Only II and III follow

 

Q. 35 If HOSPITAL is written as 32574618 in a certain code, how would POSTAL be written in that code?

A. 752618

B. 725618

C. 725168

D. 725681

 

Q. 36 Find the missing number from the given responses.

173 (24) 526

431 (18) 325

253 (?) 471

A. 22

B. 42

C. 30

D. 6

 

Q. 37 After interchanging ÷ and +, 12 and 18, which one of the following equations becomes correct?

A. (90 x 18) + 18 = 60

B. (18 +6) ÷ 12 = 2

C. (72 =18) x 18 = 72

D. (12 + 6) x 18 = 36

 

Q. 38 If SPARK is coded as TQBSL, what will be the code for FLAME ?

A. GMBNF

B. GNBNF

C. GMCND

D. GMBMF

 

Q. 39 Find out the correct answer for the unsolved equation on the basis of the given equations. If 6*5 = 91

8 * 7 = 169

10*7 = 211

then 11 * 10 = ?

A. 331

B. 993

C. 678

D. 845

 

Q. 40 A child is looking for his father. He went 90 metres in the east before turning to his right. He went 20 metres before turning to his right again to look for his father at his uncle’s place 30 metres from this point. His father was not here. From here he went 100 metres to his north before meeting his father in a street. How far did the son meet his father from the starting point?

A. 90 m

B. 100 m

C. 260 m

D. 140 m

 

Q. 41 In the following question, A stands for any of the mathematical signs at different places, which are given as choices under each question. Select the choice with the correct sequence of signs which when substituted makes the question as a correct equation.

24 D 4 D 5 D 4

A. x + =

B. = x +

C. + x =

D. = + x

 

Q. 42 What is the number missing from the third target?

5 9 15

16 29 ?

49 89 147

A. 45

B. 48

C. 51

D. 54

 

Q. 43 In a classroom, there are 5 rows, and 5 children A, B, C, D and E are seated one behind the other in 5 separate rows as follows: A is sitting behind C, but in front of B. C is sitting behind E. D is sitting in front of E. The order in which they are sitting from the first row to the last is

A. DECAB

B. BACED

C. ACBDE

D. ABEDC

 

Q. 44 Little wooden cubes each with a side of one inch are put together to form a solid cube with a side of three inches. This big cube is then painted red all over on the outside. When the big cube is broken up into the original little ones, how any cubes will have paint on two sides ?

A. 4

B. 8

C. 12

D. 0

 

Q. 45 K is a place which is located 2 km away in the north-west direction from the capital P. R is another place that is located 2 km away in the south-west direction from K. M is another place and that is located 2 km away in the north-west direction from R. T is yet another place that is located 2 km away in the south-west direction from M. In which direction is T located in relation to P ?

A. south-west

B. north-west

C. west

D. north

 

Q. 46 Find out which of the diagrams given in the alternatives correctly represents the

relationship stated in the question. Sharks, Whales, Turtles.

A. a

B. b

C. c

D. d

 

Q. 47 Refer the figure and choose appropriately.

A. a

B. b

C. c

D. d

 

Q. 48 How many triangles are there in the following figure ?

A. 11

B. 13

C. 9

D. 15

 

Q. 49 From the given answer figures, select the one in which the question figure is

hidden/embedded in the same direction.

A. a

B. b

C. c

D. d

 

Q. 50 Which answer figure is the exact mirror image of the given question figure when the mirror is held from the right at PQ ?

A. a

B. b

C. c

D. d

 

Q. 51 A concave lens always forms an image which is

A. real and erect

B. virtual and erect

C. real and inverted

D. virtual and inverted

 

Q. 52 A vitamin requires cobalt for its activity. The vitamin is

A. vitamin B12

B. Vitamin D

C. Vitamin B2

D. Vitamin A

 

Q. 53 One of the constituents of tear gas is

A. ethane

B. ethanol

C. ether

D. chloropicrin

 

Q. 54 The modulus of rigidity is the ratio of

A. longitudinal stress to longitudinal strain

B. Volume stress to volume strain

C. shearing stress to shearing strain

D. tensile stress to tensile strain

 

Q. 55 The propagation of sound waves in a gas involves

A. adiabatic compression and rarefaction

B. isothermal compression and rarefaction

C. isochoric compression and rarefaction

D. isobaric compression and rarefaction

 

Q. 56 An atomic clock is based on transitions in

A. sodium

B. caesium

C. magnesium

D. aluminium

 

Q. 57 Plasma membrane in eukaryotic cells is made up of

A. phospholipid

B. lipoprotein

C. phospholipo-protein

D. phospho-protein

 

Q. 58 Which one of the following is also called the ‘power plants’ of the cell ?

A. golgi body

B. mitochondrion

C. ribosome

D. lysosome

 

Q. 59 What is the chemical name of vinegar ?

A. citric acid

B. acetic acid

C. pyruvic acid

D. malic acid

 

Q. 60 Which of the following is not a property of heavy water’?

A. Boiling point of heavy water is lower than that or ordinary water

B. Density of heavy water is higher than that of ordinary water

C. Freezing point of heavy water is higher than that of ordinary water

D. It produces corrosion

 

Q. 61 In which of the following processes is energy released ?

A. Respiration

B. Photosynthesis

C. Ingestion

D. Absorption

 

Q. 62 Animals living in the three trunks are known as

A. arboreal

B. volant

C. amphibious

D. aquatic

 

Q. 63 Arrange the following in chronological order:

1. Tughlaqs

2. Lodis

3. Saiyids

4. Ilbari Turks

5. Khiljis

A. 1, 2, 3, 4, 5

B. 5, 4, 3, 2, 1

C. 2, 4, 5, 3, 1

D. 4, 5, 1, 3, 2

 

Q. 64 The book titled ‘The Indian War of Independence’ was written by

A. Krishna verma

B. Madame carna

C. B.G. Tilak

D. V.D. Savarkar

 

Q. 65 Who was the founder of the ‘Servants of India Society’?

A. G.K. Gokhale

B. M.G. Ranade

C. B.G. Tilak

D. Bipin Chandra Pal

 

Q. 66 The term ‘Caste’ was derived from

A. Portuguese

B. Dutch

C. German

D. English

 

Q. 67 The term “Greater India” denotes

A. Political unity

B. Cultural unity

C. Religious unity

D. Social unity

 

Q. 68 Formalised system of trading agreements with groups of countries is known as

A. trading blocks

B. trade ventures

C. trade partners

D. trade organizations

 

Q. 69 Mahatma Gandhi was profoundly influenced by the writings of

A. Bernard Shaw

B. Karl Marx

C. Lenin

D. Leo Tolstoy

 

Q. 70 Seismic sea waves which approach the coasts at greater force are known as:

A. tides

B. tsunami

C. current

D. cyclone

 

Q. 71 Depression formed due to deflating action of winds are called:

A. Playas

B. Yardang

C. Ventifacts

D. Sand Dunes

 

Q. 72 The land of maximum biodiversity is

A. tropical

B. temperate

C. monsoonal

D. equatorial

 

Q. 73 If input frequency of a full wave rectifier be n, then output frequency would be:

A. n/2

B. n

C. 3n/2

D. 2n

 

Q. 74 Heat transfer horizontally within the atmosphere is called

A. conduction

B. convection

C. absorption

D. advection

 

Q. 75 Indian Standard Time relates to

A. 75.5° E longitude

B. 82.5° E longitude

C. 90.5° E longitude

D. 0° longitude

 

Q. 76 Who is rightly called the “Father of Local Self Government” in India ?

A. Lord Mayo

B. Lord Ripon

C. Lord Curzon

D. Lord Clive

 

Q. 77 The Directive Principles of State Policy was adopted from the

A. British Constitution

B. Swiss Constitution

C. U.S. Constitution

D. Irish Constitution

 

Q. 78 Which is the second nearest star to the Earth after the Sun ?

A. vega

B. sirius

C. proxima centaur

D. alpha centauri

 

Q. 79 The two forms of democracy are

A. Parliamentary and Presidential

B. Direct and Indirect

C. Monarchical and Republican

D. Parliamentary and King

 

Q. 80 Which is an extra-Constitutional body ?

A. Language Commission

B. Planning Commission

C. Election Commission

D. Finance Commission

 

Q. 81 The Prime Minister of India is

A. elected

B. appointed

C. nominated

D. selected

 

Q. 82 Which is not an All India Service?

A. Indian Administration Service

B. Indian Police Service

C. Indian Foreign Service

D. Indian Forest Service

 

Q. 83 The forest in Sunderban is called

A. Scrub jungle

B. Mangrove

C. Deciduous forest

D. Tundra

 

Q. 84 Noise is measured in

A. watt

B. REM

C. Centrigrade

D. Decibel

 

Q. 85 Who among the following captured his third consecutive National Billiards title in the year 2009

A. Pankaj Advani

B. Devendra Joshi

C. Geet Sethi

D. Dhruv Sitawala

 

Q. 86 The Messenger Satellite launched by NASA is to study

A. mercury

B. venus

C. saturn

D. jupiter

 

Q. 87 What was the name of the ship that sank near the Paradip Port in September, 2009 causingan oil spill?

A. red rose

B. black rose

C. white rose

D. green rose

 

Q. 88 Who among the following has been honoured with the prestigious Dadasaheb Phalke Award for 2007?

A. YashChopra

B. Ustsad Amjad Ali Khan

C. Manna Dey

D. A. Nageshwara Rao

 

Q. 89 An Intelligent terminal

A. has a microprocessor, but cannot be programmed by the user

B. can process small data processing jobs, with the use of a large CPU

C. interacts with the user in English

D. cannot take data from the user

 

Q. 90 Which one of the following is not a method of estimating National Income ?

A. Expenditure method

B. Product method

C. Matrix method

D. Income method

 

Q. 91 The monetary policy is India is formulated by

A. Central Government

B. Industrial Financial Corporation of India

C. Reserve Bank of India

D. Industrial Development Bank of India

 

Q. 92 A short-term government security paper is called

A. share

B. debenture

C. mutual fund

D. treasury bill

 

Q. 93 WTO basically promotes

A. Financial support

B. Global peace

C. Unilateral trade

D. Multilateral trade

 

Q. 94 Under which market condition do firms have excess capacity ?

A. Perfect competition

B. Monopolistic competition

C. Duopoly

D. Oligopoly

 

Q. 95 Price theory is also known as

A. Macro Economics

B. Development Economics

C. Public Economics

D. Micro Economics

 

Q. 96 At present, India is following

A. Fixed exchange rate

B. Floating exchange rate

C. Pegged up exchange rate

D. Pegged down exchange rate

 

Q. 97 Who among the following won the ‘ICC Cricketer of the Year Award’ for the year 2009 ?

A. M.S. Dhoni

B. Gautam Gambhir

C. Mitchell Johnson

D. Tillakaratne Dilshan

 

Q. 98 The bats can fly in the dark because

A. they can see the objects in darkness

B. they have weak legs and are likely to be attacked by predators

C. they generate flashes of light

D. they generate ultrasonic sound waves

 

Q. 99 What changes will happen to a bowl of ice and water kept at exactly zero degree Celsius ?

A. All ice will melt

B. All water will become ice

C. No change will happen

D. Only some ice will melt

 

Q. 100 National Income is the

A. Net National Product at market price

B. Net National Product at factor cost

C. Net Domestic Product at market price

D. Net domestic Product at factor cost

 

Q. 101 The H.C.F. and L.C.M. of, two numbers are 8 and 48 respectively. If one of the numbers is 24, then the other number is

A. 48

B. 36

C. 24

D. 16

 

Q. 102 The greatest number, which when subtracted from 5834, gives a number exactly divisible by each of 20, 28, 32 and 35, is

A. 1120

B. 174714

C. 5200

D. 5600

 

Q. 103 The ninth term of the sequence 0, 3, 8, 15, 24, 35…is

A. 63

B. 70

C. 80

D. 99

 

Q. 104 (0.125 + 0.027) / (0.25 – 0.15 + 0.09) is equal to

A. 0.3

B. 0.5

C. 0.8

D. 0.9

 

Q. 105 The sum of the series (1 + 0.6 + 0.06 + 0.006 + 0.0006 + ….) is

A. a

B. b

C. c

D. d

 

Q. 106 (figure) is equal to

A. 34

B. 36

C. 38

D. 39

 

Q. 107 A number, when divided by 114, leaves remainder 21. If the same number is divided by 19, then the remainder will be

A. 1

B. 2

C. 7

D. 17

 

Q. 108 The square root of 0.09 is

A. 0.30

B. 0.03

C. 0.81

D. 0.081

 

Q. 109 The number 0. 121212…. in the P form p/q is equal to

A. 4/11

B. 2/11

C. 4/33

D. 2/33

 

Q. 110 If (3/5)³ (3/5)⁻⁶ = (3/5)²ˣ⁻¹ then x is equal to

A. -2

B. 2

C. -1

D. 1

 

Q. 111 Two numbers are in the ratio 3 : 4. Their L.C.M. is 84. The greater number is

A. 21

B. 24

C. 28

D. 84

 

Q. 112 A drum of kerosene is 3/4 full. When 30 litres of kerosene is 7 drawn from it, it remains 12 full. The capacity of the drum is

A. 120 l

B. 135 l

C. 150 l

D. 180 l

 

Q. 113 By what least number should 675 be multiplied so as to obtain a perfect cube number ?

A. 3

B. 5

C. 24

D. 40

 

Q. 114 (figure) is equal to

A. a

B. b

C. c

D. d

 

Q. 115 (figure) is equal to

A. 1/1000

B. 1/999

C. 1/99

D. 1/9

 

Q. 116 (4.41 x 0.16) / (2.1 x 1.6 x 0.21) is simplified to

A. 1

B. 0.1

C. 0.01

D. 10

 

Q. 117 If a and b are two odd positive integers, by which of the following integers is (a 4 – b 4)  always divisible ?

A. 3

B. 6

C. 8

D. 12

 

Q. 118 If a = 1 and b = 9 , then the value of (a² + b² + ab) / (a³ – b³) is

A. 2

B. -2

C. 20

D. -20

 

Q. 119 (256 x 256 – 144 x 144) / 112 equals to

A. 420

B. 400

C. 360

D. 320

 

Q. 120 If a and b be positive integers such that a² – b² = 19, then the value of a is

A. 19

B. 20

C. 9

D. 10

 

Q. 121 If the ratio of cost price and selling price of an article be as 10 : 11, the percentage of profit is

A. 8

B. 10

C. 11

D. 15

 

Q. 122 A manufacturer marked an article at Rs. 50 and sold it allowing 20% discount. If his profit was 25%, then the cost price of the article was

A. Rs. 40

B. Rs. 35

C. Rs. 32

D. Rs. 30

 

Q. 123 A shopkeeper earns a profit of 12% on selling a book at 10% discount on the printed price. The ratio of the cost price and the printed price of the book is

A. 45 : 56

B. 45 : 51

C. 47 : 56

D. 47 : 51

 

Q. 124 By selling a bicycle for Rs. 2,850, a shopkeeper gains 14%. If the profit is reduced to 8%, then the selling price will be

A. Rs. 2600

B. Rs. 2700

C. Rs. 2800

D. Rs. 3000

 

Q. 125 By selling an article, a man makes a profit of 25% of its selling price. His profit per cent is

A. a

B. b

C. c

D. d

 

Q. 126 If A’s income is 50% less than that of B’s, then B’s income is what per cent more than that of A?

A. 125

B. 100

C. 75

D. 50

 

Q. 127   1.14 expressed as a per cent of 1.9 is

A. 6%

B. 10%

C. 60%

D. 90%

 

Q. 128 Two natural numbers are in the ratio 3 : 5 and their product is 2160. The smaller of the numbers is

A. 36

B. 24

C. 18

D. 12

 

Q. 129 If 60% of A = 4 of B, then A is

A. 9 : 20

B. 20 : 9

C. 4 : 5

D. 5 : 4

 

Q. 130 Two successive price increase of 10% and 10% of an article are equivalent to a single price increase of

A. 19%

B. 20%

C. 21%

D. 22%

 

Q. 131 (Refer the image)

A. 5/12

B. 12/5

C. 5/7

D. 7/5

 

Q. 132 An equilateral triangle of side cm has its corners cut off to form a regular hexagon. Area (in cm2 of this regular hexagon will be

A. 3√3

B. 3√6

C. 6√3

D. 5√3/2

 

Q. 133 The length (in metres) of the longest rod that can he put in a room of dimensions 10 m x 10 m 5m is

A. 15√3

B. 15

C. 10√2

D. 5√3/2

 

Q. 134 If Rs. 1000 is divided between A and B in the ratio 3 : 2, then A will receive

A. Rs. 400

B. Rs. 500

C. Rs. 600

D. Rs. 800

 

Q. 135 What must be added to each term of the ratio 7 : 11, so as to make it equal to 3 : 4 ?

A. 8

B. 7.5

C. 6.5

D. 5

 

Q. 136 A sum of money at compound interest doubles itself in 15 years. It will become eight times of it self in

A. 45 years

B. 48 years

C. 54 years

D. 60 years

 

Q. 137 Buses start from a bus terminal with a speed of 20 km/hr at in tervals tervals of 10 minutes.

What is the speed of a man coming from the opposite direction towards the bus terminal if he meets the buses at intervals of 8 minutes ?

A. 3 km/hr

B. 4 km/hr

C. 5 km/hr

D. 7 km/hr


Q. 138 If the circumference of a circle is decreased by 50% then the per centage of decrease in its area is:

A. 25

B. 50

C. 60

D. 75

 

Q. 139 At what rate per cent per annum will a sum of Rs. 1,000 amount to Rs. 1,102.50 in 2 years at compound interest ?

A. 5

B. 5.5

C. 6

D. 6.5

 

Q. 140 What annual payment will discharge a debt of Rs. 6,450 due in 4 years at 5% per annum simple interest ?

A. Rs. 1400

B. Rs. 1500

C. Rs. 1550

D. Rs. 1600

 

Q. 141 The average of the first 100 positive integers is

A. 100

B. 51

C. 50.5

D. 49.5

 

Q. 142 In a family, the average age of a father and a mother is 35 years. The average age of the father, mother and their only son is 27 years. What is the age of the son?

A. 12 years

B. 11 years

C. 10.5 years

D. 10 years

 

Q. 143 If 5 men or 7 women can earn Rs. 5,250 per day, how much would 7 men and 13 women earn per day ?

A. Rs. 11600

B. Rs. 11700

C. Rs. 16100

D. Rs. 17100

 

Q. 144 If A and B together can complete a piece of work in 15 days and B alone in 20 days, in how many days can A alone complete the work ?

A. 60

B. 45

C. 40

D. 30

 

Q. 145 By walking at 3/4 of his usual speed, a man reaches his office 20 minutes later than his usual time. The usual time taken by him to reach his office is

A. 75 minutes

B. 60 minutes

C. 40 minutes

D. 30 minutes

 

Q. 146 A can complete a piece of work in 18 days, B in 20 days and C in 30 days. B and C together start the work and are forced to leave after 2 days. The time taken by A alone to complete the remaining work is

A. 10 days

B. 12 days

C. 15 days

D. 16 years

 

Q. 147 A train, 300 m long, passed a man, walking along the line in the same direction at the rate of 3 km/hr in 33 seconds. The speed of the train is

A. 30 km/hr

B. 32 km/hr

C. 328 km/hr

D. 358 km/hr

 

The pie chart, given here, represents the number of valid votes obtained by four students who contested election for school leadership. The total number of valid votes polled was 720.

Observe the chart and answer the questions based on it.

 

 

Q. 148 What was the minimum number of votes obtained by any candidate ?

A. 100

B. 110

C. 120

D. 130

 

Q. 149 What was the winner ?

A. Shivaram

B. Paramjeet

C. Yasin

D. Vishwanath

 

Q. 150 By how many votes did the winner defeat his nearest rival ?

A. 40

B. 45

C. 48

D. 50

 

Q. 151 He is a university professor (a)/ but of his three sons (b)/ neither has any merit. (c)/ No error (d).

A. a

B. b

C. c

D. d

 

Q. 152 After knowing truth,(a)/ they took the right decision (b)/ in the matter. (c)/ No error (d).

A. a

B. b

C. c

D. d

 

Q. 153 It is time you (a)/ decide on your next (b)/ course of action. (c)/ No error (d).

A. a

B. b

C. c

D. d

 

Q. 154 He who has suffered most (a)/ for the cause, (b)/ let him speak (c)/ No error (d).

A. a

B. b

C. c

D. d

 

Q. 155 A cup of coffee (a)/ is an excellent complement (b)/ to smoked salmon. (c)/ No error (d).

A. a

B. b

C. c

D. d

 

Q. 156 The hotel was not too expensive

A. was it ?

B. wasn’t it ?

C. is it ?

D. isn’t it ?

 

Q. 157 Like humans, zoo animals must have a dentist their teeth

A. fill

B. filled

C. filling

D. to be filled

 

Q. 158 It was very kind of you to do the washing-up, but you _ it.

B. hadn’t to do

C. mightn’t have done

D. mustn’t have done

 

Q. 159 He went sea alone.

A. in

B. to

C. into

D. on

 

Q. 160 The……of our civilization from an agricultural society to today’s complex industrial world was accompanied by war.

A. adjustement

B. migration

C. route

D. metamorphosis

 

Q. 161 Choose the opposite word

FLORID

A. weak

B. pale

C. monotonous

D. ugly

 

Q. 162 Choose the opposite word

VERITY

A. sanctity

B. reverence

C. falsehood

D. rarity

 

Q. 163 Choose the opposite word

PERSPICUITY

A. vagueness

B. dullness

C. unfairness

D. unwillingness

 

Q. 164 Choose the opposite word

FERVENT

A. inexcitable

B. enduring

C. dispassionate

D. subdued

 

Q. 165 Choose the opposite word

MEANDERING

A. sliding

B. sloping

C. strained

D. straight

 

Q. 166 Choose the best word

LUXURIANT

A. luxury-loving

B. lovely

C. rich

D. abundant

 

Q. 167 Choose the best word

CANTANKEROUS

A. cancerous

B. ferocious

C. quarrelsome

D. fissiparous

 

Q. 168 Choose the best word

ONUS

A. sadness

B. happiness

C. responsibility

D. criticism

 

Q. 169 Choose the best word

DERISION

A. humiliation

B. embarrassment

C. ridicule

D. condemnation

 

Q. 170 Choose the best word

TRITE

A. commonplace

B. clever

C. brief

D. impudent

 

Q. 171 Obviously he isn’t cut up to be. good teacher.

A. cut out

B. cut in

C. cur for

D. no improvement

 

Q. 172 Power got with money is the most craved for today.

A. sought after

B. wished for

C. welcomed for

D. no improvement

 

Q. 173 The brown shirt wants washing.

A. has to wash

B. is in need of a wash

C. requires wash

D. no improvement

 

Q. 174 You are asked to copy this letter word by word

A. word for word

B. word with word

C. word to word

D. no improvement

 

Q. 175 The weak man is a slave to his sensuous pleasures.

A. sensory

B. sensual

C. secondary

D. no improvement

 

Q. 176 An underhand device resorted to in order to justify misconduct

A. subterfuge

B. man-oeuvre

C. stratagem

D. complicity

 

Q. 177 Impossible to describe

A. miraculous

B. ineffable

C. stupendous

D. appalling

 

Q. 178 One who criticizes popular beliefs which he thinks is mistaken or unwise

A. philistine

B. iconoclast

C. imposter

D. cannibal

 

Q. 179 Detaining and confining someone

A. interruption

B. interrogation

C. interment

D. internment

 

Q. 180 Science of the races of mankind

A. genealogy

B. epistemology

C. ethnology

D. sociology

 

Q. 181 Find the correct word

A. collaborate

B. comemorate

C. colate

D. choclate

 

Q. 182 Find the correct word

A. circuiteous

B. clairvoyant

C. chivelery

D. cavalcade

 

Q. 183 Find the correct word

A. severety

B. sovereignity

C. superiorty

D. serenity

 

Q. 184 Find the correct word

A. cummulative

B. comemmorative

C. accummulative

D. accommodative

 

Q. 185 Find the correct word

A. benidiction

B. besmerch

C. beneficient

D. benevolence

 

Q. 186 1. Education in India had a glorious beginning.

P. But after the British rule, it faced many changes.

Q. It went on for centuries with the same glory.

R. English as the medium of instruction had a very great response.

S. One of the changes was the introduction of English as the medium of instruction.

6. As the Britishers left we had a complexity of opinions regarding English

A. PQRS

B. QPSR

C. PQSR

D. SRPQ

 

Q. 187 1. It is easy to criticize the people at the helm, for the slow progress in every field.

P. We are well aware that the intellectuals are leaving our country for better employment opportunities.

Q. Then question remains unanswered because our country cannot show opportunities to the intellectuals.

R. Then, what about their obligation to the Motherland ?

S. First, we should ask ourselves as to what is happening to the young intellectuals in India.

6. This situation of ‘Brain-Drain leads to a variety of problems.

A. PSQR

B. RPSQ

C. PSRQ

D. SPRQ

 

Q. 188 1. It is the responsibility of parents to teach the young moral values in life.

P. Many children take advantage of their parents’ busy schedule.

Q. This results in children’s ignorance of social values.

R. The reason behind it is that parents are quite busy nowadays.

S. Nowadays parents spend very meagre time with children.

6. As such, the society is going away from the value system.

A. SRPQ

B. PQRS

C. SQRP

D. SPQR

 

Q. 189 1. The man who does his duty without any selfish desire for fruit may be called a sanyasi as well as yogi.

P. The man who has achieved much evenness of temper will be serene, because his mere thoughts are changed with the strength of action.

Q. He would practise yoga, i.e., evenness of temper, and cannot but perform action.

R. The root of the matter is that one should not allow his mind to flit from object of desire to another and from that to a third.

S. But he who abstains from action altogether is only an idler.

6. A yogi is one who is not attached to his objects of sense or to action and whose mind has ceased to roam restlessly.

A. SRQP

B. RQPS

C. QRSP

D. PRSQ

 

Q. 190  1. This was an important day for at took.

P. It was a cold day, but Alatook would be warm.

Q. For the first time he was going to hunt seals alone.

R. First he put on his fur-lined jacket.

S. Then he put on mittens and boots of deerskin to protect his hands and feet from the cold.

6. Finally he picked up the gun he had cleaned so carefully the day before.

A. PQRS

B. QPRS

C. PRSQ

D. QRPS

 

Q. 191 They have made a film based on this novel.

A. A film was based on. this novel and made.

B. A film have been made based . on, this novel.

C. A film, based on this novel, has been made

D. A film has been based and made on this novel.

 

Q. 192 The people couldn’t move me to the hospital and the doctor operated on me at home.

A. I couldn’t be moved to the hospital and was operated on at home by the doctor.

B. I couldn’t be moved to the hospital and I had to be operated on at home.

C. I couldn’t be moved to the hospital and I was operated at home by the doctor.

D. I couldn’t be moved to the hospital by the people and operated on at home.

 

Q. 193 Why did he deprive you of the membership ?

A. Why you were deprived of the membership?

B. Why were you deprived of his membership by him ?

C. Why was he deprived of his membership

D. Why were you deprived of your membership by him ?

 

Q. 194 The news has been brought to us by him.

A. He brought us the news.

B. He has brought us the news.

C. He was brought the news to us.

D. We brought the news to him.

 

Q. 195 Not a word was spoken by the criminal in self-defence.

A. The criminal spoke not a word in self-defence.

B. The criminal in self-defence spoke no word.

C. The criminal did not speak a word in self-defence.

D. The criminal spoke in self defence not a word.

 

PASSAGE

Every profession or, trade, every art and every science has its technical vocabulary, the function of which is partly to designate things or processes which have no names in ordinary English and partly to secure greater exactness in nomenclature. Such special dialects or jargons are necessary in technical discussion of any kind. Being universally understood by the devotees of the particular science or art, they have the precision of a mathematical formula. Besides, they save time, for it is much more economical to name a process than to describe it. Thousands of these technical terms are very properly included in every large dictionary, yet, as a whole, they are rather on the outskirts of the English language than actually within its borders. Different occupations, however, differ widely in the character of their special vocabularies. In trades and handicrafts and other vocations like farming and fishing that have occupied great numbers of men from remote times, the technical vocabulary is very old. An average man now uses these in his own vocabulary. The special dialects of law, medicine, divinity and philosophy have become familiar to cultivated persons.

 

Q. 196 Special words used in technical discussion

A. may become part of common speech

B. never last long

C. should resemble mathematical formula

D. should be confined to scientific fields

 

Q. 197 The writer of this article is

A. a scientist

B. a politician

C. a linguist

D. a businessman

 

Q. 198 This passage is primarily concerned with

A. various occupations and professions

B. technical terminology

C. scientific undertakings

D. a new language

 

Q. 199 It is true that

A. various professions and occupations often interchange words

B. there is always a non-technical word that may be substituted for the technical word

C. the average man often uses in his own vocabulary what was once technical language not meant for him

D. everyone is interested in scientific findings

 

Q. 200 In recent years, there has been a marked increase in the number of technical terms in the nomenclature of

A. Farming

B. Fishing

C. Sports

D. Government

 

 

Answer Sheet
Question 1 2 3 4 5 6 7 8 9 10
Answer C A A C C B D B D C
Question 11 12 13 14 15 16 17 18 19 20
Answer C B D C D C B D D C
Question 21 22 23 24 25 26 27 28 29 30
Answer D A C B B A C C D C
Question 31 32 33 34 35 36 37 38 39 40
Answer B A D C B A B A A B
Question 41 42 43 44 45 46 47 48 49 50
Answer B B A B C C C B D C
Question 51 52 53 54 55 56 57 58 59 60
Answer B A D B A B A B B A
Question 61 62 63 64 65 66 67 68 69 70
Answer A A D D A A B A D B
Question 71 72 73 74 75 76 77 78 79 80
Answer B A D B B B D C A B
Question 81 82 83 84 85 86 87 88 89 90
Answer B C B D A A A C A C
Question 91 92 93 94 95 96 97 98 99 100
Answer C D D B D B C D C B
Question 101 102 103 104 105 106 107 108 109 110
Answer D B C C A B B A C C
Question 111 112 113 114 115 116 117 118 119 120
Answer C D B A B A C A B D
Question 121 122 123 124 125 126 127 128 129 130
Answer B C A B D B C A D C
Question 131 132 133 134 135 136 137 138 139 140
Answer B C B C D A C B A B
Question 141 142 143 144 145 146 147 148 149 150
Answer C B D A B C D C A A
Question 151 152 153 154 155 156 157 158 159 160
Answer C A B C D A A D B D
Question 161 162 163 164 165 166 167 168 169 170
Answer B C A C D D C C C A
Question 171 172 173 174 175 176 177 178 179 180
Answer A D C A B B B B D C
Question 181 182 183 184 185 186 187 188 189 190
Answer A B D D D B D A A B
Question 191 192 193 194 195 196 197 198 199 200
Answer C A D B C C C B C D

IBPS SO Prelims 2017 Previous Paper

IBPS SO Prelims 2017

Direction (1-5): Study the information given below and answer the questions based on it. friends A, I, C, D, E, F, G and H are sitting in a straight line facing north and southdirection. Among them four are facing south, while four are facing north. They play different games Swimming, Cricket, Kabaddi, Hockey, Tennis, Volleyball, Basketball and Chess. The following information is given about them:

1) I who plays chess sits exactly between D and the person who plays Kabaddi.
2) The person who plays tennis faces north and is to the immediate right of the person who plays Swimming.
3) C and D are not the immediate neighbours of G. They do not sit at the extreme ends of the row.
4) The person who plays hockey sits at one of the ends and is second to the right of G who faces south. A does not like to play cricket.
5) The person who plays cricket is third to the left of the person who plays volleyball.
6) Also, C does not play Kabaddi. E faces south.
7) H is second to the right of the person who plays basketball.
8) A is to the immediate right of the person who faces north.
9) Exactly one person sits between D and C. C faces south.

Question 1. The person to the immediate left of C plays which of the following games?
A. Swimming

B. Cricket

C. Tennis

D. Basketball

E. Kabaddi

Question 2. Who sits exactly between I and the person who plays volleyball?
A. G

B. D

C. H

D. F

E. None of these

Question 3. Which of the following statements is true?
A. The person second to the left of D faces north

B. Exactly one person sits between I and C

C. The person to the immediate right of A plays hockey.

D. F faces south

E. None of these

Question 4. Who plays cricket?
A. D

B. E

C. G

D. F

E. A

Question 5. Four of the following five are alike and hence form a group. Which of the following five does not belong to the group?
A. FA

B. DH

C. CE

D. IG

E. AD

Direction (6-9): Read the following information carefully and answer the questions that follow:
P, Q, R, S, T, V and W are seven different boxes of different colours i.e. black, silver, red, pink, yellow, white and green but not necessarily in the same order. Box S is immediately above yellow colour box. More than three boxes are there between pink and silver colour box. There are two boxes between the box Q and box T. There is only one box between box T and box W. There are three boxes between box W and box P, which is of black colour. There are only two boxes between box P and box R, which is of white colour. The silver colour box is immediately above the box W. More than three boxes are there between yellow and pink colour box. T is of red colour box.

Question 6. How many boxes are there between green colour box and silver colour box?
A. One

B. Two

C. Three

D. Four

E. None

Question 7. Box Q is of which colour?
A. Silver

B. Green

C. Yellow

D. Black

E. Pink

Question 8. Which box is placed immediately above red colour box?
A. P

B. S

C. R

D. T

E. W

Question 9. How many boxes are there between box W and box Q?
A. None

B. One

C. Two

D. Three

E. Four

 Direction: Read the following information carefully and answer the questions that follow:

Seven persons Tina, Vini, Yasir, Rishi, Sanya, Pankaj and Urmila live on the separate floors of a 7- floor building. Ground floor is number one; second floor is number two and so on. Each of them goes to a city viz. Paris, Istanbul, Shanghai, Durban, London, Madrid and Dubai but not necessarily in the same order.

Only three people live above the floor on which Sanya lives. Only one person lives between Sanya and the one who goes to Paris. Vini lives just below the person who goes to Madrid. Only three people live between the one who goes to Paris and London. The person who goes to Madrid lives on an even numbered floor. Urmila lives just above Rishi. Urmila does not go to London.Only two persons live between Pankaj and the one who goes to Durban. Pankaj lives above the person who goes to Durban. Yasir does not go to Istanbul. Tina does not live just above or just below Sanya. The one who goes to Shanghai does not live just above or just below Pankaj.

Question 10. Who among the following lives on floor number 7?
A. Tina

B. Yasir

C. Pankaj

D. Urmila

E. Vini

Direction (11-15): Read the following information carefully and answer the questions given below it. Seven persons A, U, S, T, J, Z and M live in four different floors. The lowermost floor is floor number 1 and the topmost floor is floor
number 4. Two persons live on each floor while only one person lives on any of the floor. They go to movies in different months viz. January, February, March and April. They go to movies either on 13th of the month or on 27th of the month. Two persons go on each month while only one person go in any of the month. A lives neither on floor 1 nor on floor 4. M goes to movie in a month which has the least number of days but not on 27th. U does not go to movie on 13th of any month. Only one person lives on floor 3 and he goes to movies in February. J and Z go to movies in the same month but not in January. They live on same floor. A does not go to movie on 13th of any month. U does not live above M. Only one person goes to movie in February. S and T go to movies in different months. S does not live with U or T. T goes in March along with A. J goes after Z. J and Z can’t be on first floor. S does not go to movie on 27th of any month. T goes on 13th of the month.

Question 11. Who goes to movies in April?
A. Z and J

B. S and U

C. T and A

D. M and S

E. T and M

Question 12. Find the odd man out.
A. Z

B. J

C. M

D. A

E. T

Question 13. When does S go to the movie?
A. 27th January

B. 13th April

C. 13th March

D. 27th April

E. 13th January

Question 14. How many persons go to movie between J and U?
A. One

B. Three

C. More than three

D. Two

E. None

Question 15. Who goes fourth to watch the movie?
A. M

B. A

C. T

D. U

E. S

×

Hello!

Click one of our representatives below to chat on WhatsApp or send us an email to info@vidhyarthidarpan.com

×